Vous êtes sur la page 1sur 77

1.

The pressure within the pleural space is negative with respect to atmospheric pressure,
except for which of the following?

A. At end expiration
B. At end inspiration
C During a Valsalva maneuver
D. If the patient has a tracheostomy
E. When taking a deep breath

2. A 30-years-old alcoholic man presents with acute severe upper abdominal pain and
vomiting. He is admitted to the intensive therapy unit with a diagnosis of severe acute
pancreatitis Forty-eight hours later he develops peripheral paraesthesiae and carpopedal
spasms. The most likely underlying metabolic abnormality is:-

A. hypercalcaemia
B. hypernatraemia
C. hypocalcaemia
D. hypokalaemia
E. hyponatraemia

3. A 50-year-old woman attends her Genera) Practitioner due to a change in appearance.


She finds difficulty removing her rings, reports: an increase ‘in shoe size and photographs
reveal a change in her facial appearance. Visual field tests are-performed to direct
confrontation. Which of the following defects is most likely to be associated with her
presentation?

A. Binasal hemianopia
B. Bitemporal hemianopia
C. Homonymous hemianopia
D. Inferior quadrantanopia
E. Unilateral loss of vision

4. A 24-year-old man falls off his motorbike, fracturing one arm, both femurs, and
rupturing- his spleen. He required surgery and 20 units of blood. Twenty-four hours after
admission he has passed 350 ml of urine. His blood pressure is 90/50 mmHg. and he has low
skin turgor. Which of the following best describes the changes in the renin-angiotensin
system initiated at the juxtaglomerular apparatus in the patient?
Renin Angiotensin II Aldosterone
Concentration concentration concentration
A ↓ ↓ ↓
B ↓ ↑ ↓
C ↑ ↓ ↓
D ↑ ↓ ↑
E ↑ ↑ ↑
5. A 35-year-old woman presents to the Emergency Department with a second attack of
painful frank haematuria. What is the most likely pathology underlying this presentation?

A. Gram negative urinary tract infection


B. Interstitial cystitis
C. Renal cell carcinoma
D. Transitional cell carcinoma of bladder
E. Urinary tuberculosis

6. 60-year-old man presents with a short history of pain in the right cheek and right upper
teeth Maxillary sinus infection is diagnosed. This sinus is particularly prone to infection
because of?

A. absence of cellia on the epithelium lining the sinus


B. closeness of the sinus to the nasal cavity
C. poor blood supply
D. Position of the sinus ostium high on the medial wall
E .relationship of the front teeth to the floor of the sinus

7. A 75-year-old insulin-dependent diabetic man has undergone a hemicolectomy. On the


first postoperative day he is nil-by-mouth, on subcutaneous insulin, maintenance IV
infusion with-Saline and intravenous morphine via patient controlled analgesia. He is
confused after a brief convulsion, has slurred speech and weakness of his right side. His
pulse is 110 beats/minute, respiratory rate 25 breaths/minute, blood pressure 160/95
mmHg and Sp02 95% on room air The most likely cause of this convulsion and confusion is:

A hypoglycemia
B. hyponatremia
C. inhibition of noradrenaline re-uptake
D. stimulation of ∂ opioid receptors
E. stimulation of GABA receptors

8. A 30-year-old woman presents to the general surgical clinic with a 1.5 cm cervical lymph node
along the anterior border of sternocleidomastoid muscle.-Clinical examination and routine blood
tests are ; unremarkable. She undergoes an excisional biopsy of the lump as a day case. The
histology report reveals Encapsulated infiltrative carcinoma with marked fibrosis and cystic
changes within the lymph node. What is the most likely primary pathology?

A. Anaplastic thyroid carcinoma


B. Follicular thyroid carcinoma
C. Malignant lymphoma of the thyroid
D. Medullary thyroid carcinoma
E. Papillary thyroid carcinoma
9. A 37-year-old man presents with severe headache, photophobia arid neck stiffness.
Karnig's sign is positive and a midline lumbar puncture is performed immediately to
determine if a cerebrospinal fluid (CSF) pathogen is involved. In performing this procedure,
which is 'he first of the following structures to be pierced by the lumbar puncture needle?

A. Anterior longitudinal ligament


B. Arachnoid mater
C. Dura mater
D Interspinous ligament
E. Posterior longitudinal ligament

10. A 26-year-old woman presents with severe headache, photophobia and neck stiffness.
Among other investigations, a lumbar puncture is earned out and cerebrospinal fluid is
sent for microscopy and culture. Culture shows a growth of Neisseria meningitidis. What is
likely to be the predominant-cell type in the cerebrospinal Fluid?

A. Basophils
B. Eosinophils
C. Lymphocytes
D. Macrophages
E Neutrophils

11. A 70-year-old woman from a nursing home presents to the! Emergency Department
with abdominal pain and vomiting. On examination, she is dehydrated and her abdomen is
distended. There is a 3 cm 4 cm swelling in her right groin which is non-tender, and there is
no cough impulse. At operation, a femoral Hernia found. Which of the following lies
medial to the neck of the hernia?

A. Femoral artery
B. Femoral nerve
C. Femoral vein
D Lacunar ligament
E. Pectineal ligament

12. A 26-year-old man presents to the Emergency Department with extensive bleeding
from his arm after sustaining a glass injury. On examination there is a 7 cm transverse
laceration across the anterior aspect of his elbow. On exploring the cubital fossa you would
expect the brachial artery to be:

A. anterior to the median nerve


B. lateral to the biceps tendon
C. lateral to the median nerve
D. medial to the median nerve
E. superficial to the bicipital aponeurosis
13. A 52-year-old man is found to have multiple myeloma. What skull X-ray appearances
would be characteristic?

A. Diffuse thickening of the calvarium


B. Hair on end appearance.
C Multiple fractures
D Multiple osteolytic lesions
E. Multiple osteosclerotic lesions

14. A 50-year-old woman presents with a history of right upper quadrant pain and
jaundice. She reports that her urine was dark in colour and that her stools are offensive
and difficult to flush Which of the following explains the dark urine?

A. increase in conjugated bilirubinuria


B. Increase in unconjugated bilirubinuria
C. Increase in urea excretion
D. Increase in urinary urobilinogen
E. Reduced enterohepatic bile salt circulation

15. A 45-year-old mart presents with backache and leg pain due to a prolapsed lumbar
Intervertebral disc. The pain, which is aggravated by coughing and sneezing, radiates to
the lateral aspect of the foot. On examination, there is weakness of the plantar flexors of
the foot, which nerve root is most likely to be involved?

A. T12
B. L4
C. L5
D S1
E. S2

16. A 62-year-old woman presents to her General Practitioner with a two-week history of
backpain. She has lost 8 cm in height over the last five years Investigations reveal:

Findings Normal
Adjusted ( corrected) Calcium 2.78 mmol/L 2.15-2.55 mmol/L
Phosphate 0.84 mmol/L 0.8-1.4
Estimated GFR 0.96ml/minute More than 90
PTH 8.9 pmol/L 0.95-5.7
Which of the following is the most likely cause?

A Hypoparathyroidism
B Primary hypemarathyroidism
C Pseudo hypoparathyroidism
D Secondary hyperparathyroidism
E Tertiary hyperparathyroidism
17. A 21-year-old man fractures his ulna and radius. He is treated by means of a plaster
cast, which is removed after four weeks. What pathological process is most likely to have
affected the immobilized muscles?

A. Aplasia
B Atrophy
C Hyperplasia
D Hypertrophy
E Neoplasia

18. A 22-year-old man has been stabbed in the left fifth intercostal space at the edge of the
sternum. Which structure is most likely to have been penetrated?

A Left atrium
B Left lobe of the liver
C Left ventricle
D Right atrium
E Right ventricle

19. An 80-year-old man dislocates his shoulder it is reduced in the Emergency Department.
At review three weeks later, he is unable to actively initiate abduction of his arm. Which
muscle most likely damaged?

A Clavicular head oi pectoralis major


B Infraspinatus
C Supraspinatus
D Teres major
E Trapezius

20. An 8-year-old boy is admitted for assessment of rectal blood loss. His mother describes
this as bright red blood in the toilet pan. He has no pain on defalcation. There is no family
history. What is the most likely cause?

A. Adenomatous polyp
B Familial adenomatous polyposis
C Juvenile polyp
D Metaplastic polyp
E. Peutz-Jegher's syndrome

21. A 48-year-old woman presents with abdominal pain and five to six episodes of bloody
diarrhoea each day. She also complains of pain- in the knees and elbows and recent visual
problems. What is the most likely diagnosis?

A. Collagenous colitis
B. Diverticulitis
C. Infective colitis
D. intestinal tuberculosis
E Ulcerative colitis

22. A 56-year-old man presents with acute epigastric pain and vomiting. On examination
he has guarding in the upper abdomen. Investigations reveal:
Value Normal
Serum amylase 900 IU/L ≤100
Serum ALT 61 IU/l ≤50
Alkaline Phosphatase 98 IU/l 20-120
Albumin 38 g/L 35-50
Gamma GT 76 IU/L ≤60
Bilirubin 15 umol/L 0-20
Triglyceride 1.6 mmol\L ≤1.7
Corrected Calcium 3.30 mmol\L 2.20-2.70
Which of the following is the most likely etiology of this condition?

A. Alcohol
B Choledocholithiasis
C Hyperparathyroidism
D Hypertriglyceridaemia
E Mumps

23. A 30-year-old man is admitted to the intensive care unit with an isolated severe head
injury. A CT "can shows multiple intracerebral bleeds but no midline shift. He is intubated
and ventilated. His pupils are dilated and react sluggishly to light. His heart rate is 50
beats/minute blood pressure 170,110 mmHg and his respiratory rate is set at 10
breaths/minute. The rising blood pressure is likely to be caused by:

A. aortic and carotid baroreceptor stimulation


B Cortisol stimulation
C. renin-angiotensin stimulation
D. sympathetic stimulation related to blood loss
E. sympathetic stimulation related to intracranial pressure

24. A 42-year-old multiparous woman is admitted to the Emergency Department due to


pelvic discomfort. The duty gynecologist diagnoses uterine prolapse. Which anatomical
structure gives significant direct support to the uterus?

A. Cervical ligaments
B. Mesometrial part of the broad ligament
C Mesosalpingial part of the broad ligament
D. Round ligaments
E. Transverse perinea! Muscles
25. An 80-year-old man is admitted to the surgical admission unit with central abdominal
pain. His blood pressure is 100/60 mmHg and his pulse is 110 beats/minute with a
respiratory rate of 25 breaths/minute. He is known to have severe chronic obstructive
pulmonary disease (COPD) and has been a heavy smoker all his life. On examination he has
a rigid abdomen Arterial blood gases show:
PH pO2 pCo2 Bicarbonate Base Excess
Finding 7.24 9 kPa 5.3 kPA 17 mmol/L -3
Normal 7.35-7.45 11.9-13.3 4.7-6.0 22-26 -2 to +2
What is the most likely diagnosis?

A Hypovolemic shock complicated by COPD


B Metabolic acidosis due to COPD
C Metabolic acidosis due to peritonitis
D Respiratory-acidosis due to COPD
E Respiratory acidosis due to peritonitis

26. In describing the sino-atrial node, which one of the following statements is correct?

A it is part of the somatic nervous system


B It is usually supplied by the left coronary artery
C If lies in the wall of the left atrium
D It lies in the wall of the right ventricle
E It receives fibers derived from the vagus nerves

27. A fit 21-year-old man is admitted with an acute abdomen, subsequently diagnosed as
gastroenteritis. As part of the host immune response, which of the following
immunoglobulins and cells are correctly paired?

A. IgA-macrophage
B IgD-small bowel epithelium
C IgE-endothelium
D IgG-plasma cell
E IgM thymocyte

28. A 45-year-old man presents with fever and pain in his right loin and groin. A soft
swelling was noted in his femoral triangle. Diagnosis of a psoas abscess was made. Which
of the following statements is most accurate regarding psoas major?

A. It arises from the lateral borders of the bodies of T12 to L5 •


B. It extends the hip
C. It inserts into the greater trochanter of the femur
D. It is innervated from T12 and LI
E. It passes posterior to the capsule of the hip joint
29. Which one of me following muscles is an extensor of the hip?

A. Adductor longus
B. Gracilis
C. Iliopsoas
D. Pectineus
E. Semitendinosus

30. A tall, thin, 25-year-old woman presents to the surgical clinic with a complaint of
swelling in the front of her neck for the last four months. On clinical examination she has a
swelling in the left lobe of the thyroid and multiple neuromatous lesions within the oral
cavity. Her blood pressure is 220/120 mmHg. Laboratory investigations show that her
calcium and electrolytes are normal and serum calcitonin and urinary VMA levels are
elevated, Which of the following is the most likely cause of the hypertension?

A Conn's syndrome
B Cushing's disease
C Essential hypertension
D Phaeochromocytoma
E Renal artery stenosis

31. A 65-year-old man presents with haematuria and left loin pain. Computerised
tomography demonstrates a left renal tumour. He undergoes a left radical nephrectomy.
Where does the left renal artery lie?

A. Anterior to the left gonadal vein


B Anterior to the left renal vein
C Anterior to the splenic vein
D Posterior to the left renal vein
E Superior to the superior mesenteric artery

32. Within the posterior triangle, which nerve is at particular risk of damage during
surgery?
A. Auricular branch of the facial
B Glossopharyngeal
C Phrenic
D Recurrent laryngeal
E Spinal accessory

33. The right and left pulmonary arteries are derived from which of the following
embryological aortic arches?
A Second aortic arch
B Third aortic arch
C Fourth aortic arch
D fifth aortic arch'
E Sixth aortic arch
34. A 47-year-old man presents to the Emergency Department six hours after a climbing
accident initial assessment suggests that he had lost 1 L of blood. His blood pressure is
105/70 mmHg and his pulse rate is 88 beats/minute. He is catheterised and his bladder
contains 120 ml of urine. Activation of which of the following transport systems best
describes how aldosterone leads to maintenance of the intravascular volume and oliguria
in this men?

A. Na+/glucose in the proximal tubule


B. Na/H+ in the descending loop of Henle
C. Na+/K in the ascending loop of Henle
D. Na+/Cl in the distal tubule
E. Na/K in the collecting ducts

35. During arch aortography, the vertebral artery would be seen to arise directly from
which of the following?
A. Arch of the aorta
B Brachiocephalic artery
C Common carotid artery
D Internal carotid artery
E Subclavian artery

36. A 65-year-old man, remains, on the high dependency unit following a gastrectomy,
three days previously. His urine output: has averaged 80ml/hour for the last 24 hours. He
has one dry abdominal drain arid no nasogastric losses: His urea and electrolytes are
normal He is to stay nil by mouth for a further 24 hours. Which of the following fluid
regimes is most appropriate to his electrolyte and water requirements over the next 24
hours?

A. 1 L of 0.9 % saline plus 1.5 L. of 4 % dextrose/0.18 % saline


B. 1 L of 0.9 % saline plus 1.5 L of 5 % dextrose
C. 1 L of 0.9 % saline plus 1.5 L of Hartmann's solution
D. 2 L of 0.9% saline
E. 2 L of 4 % dextrose/0,18 % saline

37. A 3-year-old boy is admitted to hospital with severe vomiting. Radiographic


examination reveals that he is suffering from annular pancreas. Which of these structures
is constricted?
A First part of duodenum
B Second part of duodenum
C Third part of duodenum
D Proximal jejunum
E Pylorus of stomach
38. A 20-year-old man with a severe head injury is being ventilated using intermittent
positive pressure. Which of the following is a physiological consequence of this?
A Decreased extracellular fluid volume
B Decreased cardiac preload
C Increase in intrathoracic blood volume
D Increased cardiac preload
E Increased cardiac stroke volume

39. A 36-year-old man presents with a two-month history of low back pain radiating to his
left leg After clinical examination he is referred for an MRI scan. This shows a prolapsed
intervertebral disc. The clinical signs are consistent with pressure on SI roots. Which
combination of clinical signs is most likely?

A Weak ankle plantarfiexion; altered sensation on the dorsum of the foot; normal ankle jerk reflex
B Weak ankle plantarfiexion; altered sensation on the sole of the foot; loss of ankle jerk reflex.
C Weak hallux dorsiflexion; altered sensation on the anterior surface of the leg; loss-of ankle jerk reflex.
D. Weak hallux dorsiflexion; altered sensation on the dorsum of the foot; normal ankle jerk reflex.
E. Weak hallux dorsiflexion; altered sensation on the sole of the foot; loss of knee jerk reflex

40. 55-year-old woman undergoes a cholecystectomy; The gallbladder contains multiple


dark irregular small stones. The most likely, cause of this type of gallstone is an increased
blood level of:

A Bile acids
B calcium
C cholesterol
D conjugated bilirubin
E unconjugated bilirubin
41. A 55-year-old man presents with a deep venous thrombosis. His full blood count shows
a haemoglobin of 18.3 g/dL (normal 13.5-17.5), a white cell count of 8 x 109/L (normal 3.9-
10.0 x 109) and a platelet count of 200 x 109/L (normal 150-400 x 109) This hematological
picture is most likely to be associated with:

A bronchial carcinoma
B pancreatic carcinoma
C prostatic carcinoma
D renal cell carcinoma
E transitional cell carcinoma of the kidney

42. A 27-year-old has had pancolitis for the last five years. Which one of these
complications is he least likely to develop?
A. Arthritis
B. Cholangitis
C Iritis
D Polyarteritis
E Toxic megacolon
43. A surgeon makes a Pfannenstiel incision for access to the pelvic organs. He incises
theabdominal wall down to and through the rectus sheath.-He retracts the rectus
abdominis muscles laterally from the midline to expose the:
A. linea alba
B. peritoneum
C. posterior rectus sheath
D. transversalis fascia
E. transversus abdominis muscle
44. A 25-year-old man undergoes splenectomy following blunt abdominal trauma. Three
days postoperatively venous blood testing is likely to reveal:
A. high platelet count
B. high red cell count
C. low platelet count
D. low while cell count
E. low red cell count

45. A 46-year-old woman is seen in the pre-admission clinic. She had a myocardial
infarction two years ago. On examination she is in atrial fibrillation and a recent
echocardiogram shows that she has a dilated left ventricle: She is on warfarin and her INR
is 6.1. Warfarin inhibits which of the following?

A. Antithrombin
B. Kallikrein
C Plasminogen
D Prothrombin
E. von Willebrand's factor

46. A 75-year-old man is admitted for elective repair of an abdominal aortic aneurysm.
Following the operation his systolic blood pressure falls below 90 mmHg and during
resuscitation he is given an infusion of fluids. There is a choice of colloid or crystalloid.
Which of the following best describes why the intensive therapy unit registrar decides to
administer a colloid?
A Colloids are not freely filterable across semi-permeable membranes
B Colloids can prevent pulmonary oedema by inducing fluid flow out of the interstitial space
C Colloids decrease the transcapillary oncotic pressure gradient
D Colloids expand the plasma volume by 200 ml for each liter infused
E Colloids migrate from the intravascular space

47. A 35-year-old man develops septicaemia following surgery for perforated acute
appendicitis He is hypotensive. Arterial blood gas analysis reveals:
PH pO2 pCo2 Bicarbonate
Finding 7.25 7.2 kPa 7.5 kPA 17 mmol/L
Normal 7.35-7.45 11.9-13.3 4.7-6.0 22-26
What is the most likely explanation for these readings?
A Compensated metabolic acidosis
B Compensated respiratory acidosis
C Mixed metabolic and respiratory acidosis
D Uncompensated metabolic acidosis
E Uncompensated respiratory acidosis

48. A.20-year-old man presents to the Emergency Department with a stab injury to the
thenar eminence. On examination he is found to have a 2 cm long laceration with loss of
sensation in the thumb and index finger and weakness: of the thenar muscles. Which of
the following structures is most likely to have been injured?

A. Anterior interosseous nerve


B. Recurrent branch of median nerve
C. Sensory and motor branches of median nerve
D. Sensory and motor branches of radial nerve
E. Sensory and motor branches of ulnar nerve

49. A 65-year-old man presents with vomiting and .weight loss. On examination he had 3
palpable epigastric mass. The transpyloric plane lies at which; vertebral level?

A T11
B T12
C L1
D L2
E L3
50. A 30-year-cid motorcyclist is brought to the Emergency Department after a road traffic
accident. He has an open fracture of the right femur. On arrival he is tachypnoeic and.
confused, with cold and clammy skin. Which of the following physiological changes is most
likely to occur?

A. Decreased reabsorption of water from the collecting tubules


B Decreased serum bicarbonate level
C Increased pH of blood
D. Increased secretion of sodium in the urine
E. increased synthesis of glycogen in the liver

51. In an emergency situation what is the most appropriate surgical method of obtaining
an airway?
A Cutting and retracting the cricothyroid muscle
B Cutting the thyrohyoid membrane .
C Dividing the thyroid isthmus
D Entering the trachea at the C7 level
E Piercing the cricothyroid membrane
52. A 75-year-old man presents with hepatomegaly and ascites. Computerised
tomography shows evidence of post hepatic portal hypertension. The inferior vena cava
passes through the Diaphragm at which vertebral level?
A. T8
B. T9
C. T10
D T11
E T12

53. A 20-year-old woman presents acutely with abdominal pain, imaging reveals a right
ovarian cystic lesion which is excised. Histoiogicai examination shows a cyst lined by
keratinizing stratified squamous epithelium. Areas of fat, muscle, thyroid and neural tissue
are seen in the wall. What is the appropriate pathological designation for this lesion?

A Adenocarcinoma
B Cystadenoma
C Dysgerminoma
D Squamous cell carcinoma
E Teratoma
54. A 25-year old woman who is two months post-partum, presents with a tender mass in
the left breast. She is pyrexial with a raised white cell count and a raised C-reactive protein
level.-A diagnosis of lactational mastitis with abscess is made. Which of the following is
the most likely causative organism?

A Aeromonas hydrophilia
B Escherichia coli
C Streptococcus' miileri
D Lactobacillus caseii
E Staphylococcus aureus

55. A 62-year-oid alcoholic man is admitted with severe acute pancreatitis. Three days
after admission he becomes hypoxic (pOa is 7.5 kPa) and confused. Chest X-ray shows a
uniform opacification of both lung fields. Which of the following is the most likely clinical
diagnosis?

A Acute left ventricular failure


B Adult respiratory distress syndrome
C. Bilateral bronchopneumonia
D. Bilateral lobar pneumonia
E. Massive pulmonary embolism

56. A 58-year-oid woman presented to the Emergency Department with a large fluctuant
swelling the site of a recent insect bite. She is anxious, tachycardic, and pyrexial. An ECG
shows critical fibrillation. She is noted to have a goiter. The swelling at the site of the bite
requires surgical drainage. Which of the following classes of drug would be most
appropriate as part of her preoperative preparation for surgery?
A. Alpha-1 adrenoceptor agonist
B Alpha-2 adrenoceptor agonist
C Alpha adrenoceptor blocker
D Beta adrenoceptor agonist
E Beta adrenoceptor blocker

57. A nulliparous 30-year-old Woman presents with a recurrent painful red area in the
areola. She gives a history of smoking cigarettes. The lesion is excised and the histology
shows squamous metaplasia of lactiferous ducts. Microbiological culture does not grow
any organism. Which of the following is the most likely diagnosis?

A Breast abscess
B Carcinoma of the breast
C Fibroadenoma
D Galactorrhea
E Subareolar abscess

58. A 65-year-old man undergoes right hemicolectomy; On the second postoperative day
he is afebrile, nauseous has a distended abdomen but very little pain. He is on intravenous
morphine via a patient-controlled analgesia pump. Plain X-ray confirms distended loops of
small intestine. The physiologic basis of postoperative paralytic ileus involves:

A. stimulation of dopaminergic receptors


B. stimulation of reversed migrating motor complexes
C. stimulation of p receptors
D. suppression of muscarinic receptors.
E. suppression of motilin due to starvation

59. A 42-year-old woman has a cholecystectomy arid develops a self-limiting


postoperative wound infection. By what process would bacterial ingestion have been
enhanced?

A Apoptosis
B Autophagy
C Metaplasia
D Opsonisation
E Phagocytosis
60. A 60-year-old non-smoker presents with three-month history of loss of weight, malaise
and breathlessness. On examination he has left supraclavicular lymph node enlargement.
Chest radiograph reveals multiple bilateral opacities. What abnormality would a biopsy of
a left supra-clavicular lymph node most probably show?
A. Chronic inflammatory cells
B. Granuloma
C. Langerhan's type giant cells
D. Multiple abnormal mitotic figures in cells
E. Reed Sternberg cells
61. A 78-year-old woman with emphysema receiving 28% oxygen by mask has the
following blood gas results The most likely interpretation is:
PH pO2 pCo2 Bicarbonate Base Excess
Finding 7.28 6.2 kPa 8 kPA 36 mmol/L +5
Normal 7.35-7.45 11.9-13.3 4.7-6.0 22-26 -2 to +2
A. mixed respiratory and metabolic acidosis
B. partially compensated metabolic acidosis
C. partially compensated respiratory acidosis
D. uncompensated metabolic acidosis
E. uncompensated respiratory acidosis

62. A 2-year-old child is referred because of an impalpable testis in the left scrotum. Which
of the following positions describes an ectopic testis?
A At the deep inguinal ring
B At the root of the penis
C At the superficial inguinal ring
D In the inguinal canal.
E in the upper part of the scrotum
63. A 60-year-oid man is found to have a 2 cm diameter mass in the upper lobe of his left
lung.The lesion is excised and it is found that the lesion includes connective tissue, mature
cartilage and ciliated epithelium. What is the most likely diagnosis?
A. Adenoma
B. Carcinoma
C. Chondroma
D. Hamartoma
E. Sarcoma

64. A 23-year-oid man presents three months following extraction of carious teeth with a
plaque like infiltration over the upper jaw with sinus formation. Yellow granules are
present in the discharging pus. What is the most likely causative organism?
A Actinomyces
B. Aspergillus
C. Blastomyces
D. Cryptococcus
E. Histoplasma

65. An 82-year-old man has complete occlusion of his inferior mesenteric artery on
angiography but no symptoms or signs of colonic ischemia. Which of the following arteries
is the most likely additional source of blood supply to the territory of the inferior
mesenteric artery?

A Left colic
B Left gastroepiploic
C Middle colic
D Splenic
E Superior rectal
66. An intravenous drug abuser is having an echocardiogram for suspected endocarditis.
Closure of the tricuspid valve occurs at the onset of which phase of the cardiac cycle?

A. Atrial systole
B Isovolumetric contraction
C Isovolumetric relaxation
D Rapid ejection
E Rapid ventricular filling

67. After being stabbed in the the left groin, a previously healthy 20-year-old man was
admitted to The emergency Department. He developed hemorrhagic shock. During
resuscitation, 10 units of type O negative red cells and 6 L of colloid were administered;
Twenty-four hours later, he noted to have severe dyspnoea for which he required
intubation and ventilation on the intensive care unit. Which of the following would be the
most likely cause of his respiratory insufficiency?
A. ABO blood incompatibility
B. Adult respiratory distress syndrome (ARDS)
C. Congestive cardiac failure
D. Tension pneumothorax
E. Viral pneumonitis
68. An 80-year-old man with a history of hypertension, controlled with thiazide diuretics.
undergoes transurethral resection of prostate under general anaesthetic. The surgery.is
prolonged and in the recovery room he complains of nausea and a headache. He later
becomes agitated and confused. Which of the following is the most likely diagnosis?

A Hypercalcaemia
B Hyperuricaemia
C Hypoglycaemia
D Hypokalaemia
E Hyponatremia

69. A 35-year-old man is admitted to hospital with vomiting, nausea and severe
headaches. An MRI scan reveals a tumour of the cerebellopontine angle. Which one of the
following pairs of cranial nerves is most likely to be compressed by this tumour?
A Accessory and vagus
B Facial and vagus
C Facial and vestibulocochlear
D Glossopharyngeal and vestibulocochlear
E Vagus and vestibulocochlear

70. After resection of d rectal tumour a patient experiences erectile dysfunction. Which of
the following nerves is most likely to have been damaged in surgery?

A Genitofemoral nerve
B Lumbosacral plexus
C Pelvic splanchnic nerves
D Perineal branch of S4
E Pudendal nerve
71. A 45-year-old woman presents with pain, swelling and stiffness affecting all her
metacarpophalangeal joints. She has noticed weakness in her grip and her handwriting has
changed. She is unable to extend her fingers fully. The index, middle and ring fingers in
both hands show an ulnar deviation. The most likely diagnosis is:
A Dupuytren's contracture
B gouty arthritis
C osteoarthritis
D radial nerve palsy
E rheumatoid arthritis
72. A 22-year-old man arrives to the Emergency Department with sudden breath less ness
due to a large pneumothorax. A chest drain is inserted into the fifth left intercostal space
in 5th mid-axillary line. There is haemorrhape into the drainage bottle. Which of the
following structures is the most likely cause of this acute haemorrhage?
A Intercostal artery
B Left pericardiophrenic artery
C Lingula of the lung
D Right ventricle of the heart
E Spleen
73. 65-year-old woman collapses after a total hip replacement. A pulmonary embolism is
suspected. Which of the following electrocardiogram changes would support this
diagnosis?

A Dominant R wave in V6
B Left axis deviation
C Left bundle branch block
D ST elevation in V1-V3 /
E T wave inversion in V1-V3

74. 40-year-old woman had the anterior lobe of the pituitary removed because of a
tumour. Without postoperative supplements, which of the following could occur?

A Failure to produce adequate amounts of thyroxine


B Fail to produce parathyroid hormone in response to hypocalcaemia
C Failure to secrete catecholamine in response to stress
D Failure to secrete insulin in hyperglycemia
E inability to concentrate urine in response to water deprivation

75. A 38-year-oid man in end-stage renal failure resulting from polycystic kidney disease
receives a cadaveric renal transplant. Good renal function is established but four weeks
later deteriorates, the serum creatinine rising by 25%. Which of the following processes is
most likely to be responsible for this deterioration?

A B-cell mediated rejection


B Circulating immune complex disease
C IgG antibody mediated rejection
D Post-transplant lymphoproliferative disorder
E T-cell mediated rejection

76. In order to expose the right axillary artery, a transverse skin incision is typically made
below the clavicle from a point just lateral to the sternal end of the clavicle to the
deltopectoral groove. Which of the following structures would be encountered in the
dissection down to the vessel?

A Lateral thoracic artery


B Phrenic nerve
C Suprascapular artery
D Thoracic duct
E Thoraco-acromial artery

77. A 4-year-old boy presents to the Emergency Department with a two-day history of
headache vomiting and drowsiness. A CT scan reveals dilatation of both his lateral
ventricles and his third cerebral ventricle. His fourth ventricle was of normal size. It is
suspected that he has an obstruction to his cerebrospinal fluid flow. At which of the
following sites is the obstruction most likely to be?
A Cerebral aqueduct (of Sylvius)
B Infundibular recess
C Interventricular foramen (of Monro)
D Lateral foramen of fourth ventricle (foramen of Luschka)
E Median foramen of fourth ventricle (foramen of Magendie)
78. A 12-year-old boy presents to the Emergency Department two hours after helping his
father cut the grass. He complains of rhinorrhea, itchy eyes, sneezing and a blocked nose.
He is apyrexial with a haemoglobin of 12.2 g/dl and white blood cell count of 6.8 * 109/L
with a raised eosinophil count. Chest X-ray is clear. Which immunoglobulin is most likely to
cause this reaction?

A IgA
B IgD
C IgE
D IgG
E IgM

79. A 35-year-old woman undergoes gastric bypass surgery for morbid obesity. At a
subesequent surgical clinic review she complains of dizziness, sweating, palpitation and
collapsing episodes after big meals without any vomiting or pain. she is otherwise well and
has no medical problems. What is the most likely explanation for her symptoms?
A. Operative denervation of stomach
B. Release of gastrin
C. Release of glucagon
D. Release of insulin
E. Vasovagal syncope
80. A 56-year-old man with chronic emphysema is on the high dependency unit (HDU), ten
days after anterior resection. He has developed acute shortness of breath' and
hypotension): Which of the following landmarks would be the most appropriate to use to
obtain a femoral arterial blood gas sample?

A. A point midway between the anterior superior iliac spine and the pubic tubercle
B. A point midway between the anterior superior iliac-spine and the' pubic symphysis
C. A point midway between the greater trochanter and the pubic symphysis
D. A point 1 cm inferior and 4 cm lateral to the pubic tubercle
E. A point 2 cm lateral to the midway point between the anterior superior iliac spine and
pubic symphysis

81. A 70-year-old man complains of persistent numbness since.an inguinal hernia repair
one year previously. The numbness affects the top of the scrotum, root of penis, and a
small area below the medial part of the inguinal ligament. Which nerve is likely to have
been damaged?
A. Femoral branch of the genitofemoral nerve
B. Genital branch of the genitofemoral nerve
C. lliohypogastric nerve
D Ilioinguinal nerve
E. Medial cutaneous branch of the femoral nerve

82. A 12-year-old girl has a diastolic murmur. It is maximally audible in the second left
intercostal space. Pathology of which structure is the most likely cause?
A. Aortic valve
B. Ductus arteriosus
C. Mitral valve
D. Pulmonary valve
E. Tricuspid valve

83. A 34-year-old pregnant woman develops a swollen leg. Her mother and maternal aunt
also had similar problems during their pregnancies. Which of the following tests is likely to
be positive?
A. Antiendomysial antibodies
B. Antimitochondrial, antibodies
C. Antinuclear antibodies
D Antiphospholipid antibodies
E. Antithyroglobuiin antibodies

84. The "fight or flight", response produces a release of epinephrine (adrenaline). What is
the primary metabolic effect of epinephrine?

A. Alanine shunt activation


B. Cortisol release
C. Glycolysis
D Tachycardia
E Vasoconstriction

85. A 58-year-old-man underwent an emergency appendectomy. Which of the following


physiological parameters are consistent with sepsis?

Heart Rate Systemic Vascular Cardiac


Resistance output
A ↓ ↓ ↑
B ↑ ↑ ↓
C ↑ ↓ ↑
D ↓ ↑ ↓
E ↑ ↓ ↓

83. A 50-year-old man with a 30-year history of pancolitis undergoes surveillance


colonoscopy which reveals a plaque-like lesion in the descending colon. Biopsy reveals a
pre-malignant change. What is the name of this pre-malignant change?
A Anaplasia
B Dysplasia
C. Hyperplasia
D. Metaplasia
E. Neoplasia

87. A 60-year-old man undergoes cystectomy for a bladder carcinoma. During surgery, the
ureters are identified. On which region of the bladder do the ureters pierce the bladder
wall?
A. Anterior surface
B Apex
C Lateral surfaces
D Neck
E Posterior surface

88. A year-old patient in the intensive care unit has a tracheostomy performed via the
second, third and fourth tracheal rings. Which intervening structure is most likely to
require transection?

A Anterior jugular vein


B Inferior thyroid veins
C Sternothyroid muscle
D Thymus
E Thyroid isthmus
89. A 23-year old man is assaulted and sustains a stabbing injury to the left groin. The
wound- is cleaned and a simple dressing applied; Six months later the patient returns with
pulsatile swelling at the site of the injury. The most probable:
lesion present is:

A Abscess
B False aneurysm of the femoral artery
C Lymphocoele
D Mycotic aneurysm of the femoral artery
E. Saphena varix

90. A 72-year-old man with a body mass index of 18.4 has not eaten for four days following
the removal of an adenocarcinoma from his descending colon. His urea is found to be 12
mmol/L (normal 3.2-7.5) and creatinine 346 umol/L (normal 35-110). A blood gas profile is
ordered. The most likely set of results would be:

PH pO2 pCo2 Bicarbonate


A 7.23 13.6 kPa 3.8 kPA 13.5 mmol/L
B 7.30 8.8 kPa 8.3 kPa 30.5 mmol/L
C 7.36 11.5 kPa 5.3 kPa 30.5 mmol/L
D 7.43 16.4 kPa 3.7 kPa 21.0 mmol/L
E 7.49 12.6 kPa 3.9 kPa 24.0 mmol/L
Normal 7.35-7.45 11.9-13.3 4.7-6.0 22-26

91. A 16-year-old boy is hit on the left side of the face by a ball. There are no broken bones
but the boy complains of numbness of the face below the eye. Which nerve has most likely
been compromised?
A Abducens nerve.
B Facial nerve
C Glossopharyngeal nerve .
D infra-orbital nerve
E Zygomatic nerve

92. A 74-year-old man, who has undergone emergency major abdominal surgery two-days
previously, is noted to be confused. He has been on furosemide for mild heart failure. The
plasma sodium is 122 mmol/L Inspection of the fluid chart shows that he has been written-
up for 1L, four-hourly intravenous 5% glucose infusions. What is the most likely cause for
the hyponatremia?
A. An ACTH (Adrenocorticotrophic hormone) response to surgery
B. Excessive sodium-free intravenous fluid administration
C. Osmotic effect of hyperglycemia induced by glucose infusions
D. Syndrome of inappropriate antidiuretic hormone
E. Use of loop diuretic in the long term
93. The Cori cycle is important in lactate metabolism in the Septic surgical patient It is used
to describe a pathway in which glucose is metabolised anaerobically to lactate in one
tissue and the lactate is converted back to glucose in another; Which one of the following
relies on this cycle to meet all of its energy needs?

A. Erythrocyte
B. Hepatocyte
C. Leukocyte
D. Osleocyte
E. Pneumocyte

94. After receiving an intermascular Injection in the buttock, a 25 year-old man complains
of inability to evert his foot Which nerve is most likely to have been injured?

A Common peroneal (fibular) component of sciatic


B Inferior gluteal
C Pudendal
D Superior gluteal
E Tibial component of sciatic

95. A 73-year-old man with a history of irregular bowel movements presents with dysuria,
pneumaturia and an Escherichia ccli urinary tract Infection. CT scans show a mass involving
the sigmoid colon and the bladder. What is the commonest cause of this presentation?

A. Adenocarcinoma of the sigmoid colon


B Colonic diverticular disease
C Crohn's disease
D Transitional cell carcinoma of the bladder
E Ulcerative colitis

96. A 7 year-old woman is in the recovery area arid receives 28% oxygen by mask. Blood
Gas Shows
PH pO2 pCo2
Finding 7.2 12 kPa 10 kPA
Normal 7.35-7.45 11.9-13.3 4.7-6.0
Reduced sensitivity of which receptors is most likely to be responsible for this blood gas
picture?

A. Adrenergic receptors
B. Baro receptors
C Central chemo-receptors
D J receptors
E Lung stretch-receptors
97. A 76-year-old woman falls and sustains an inter-trochanteric fractured neck of femur
Following operative fixation her recovery is prolonged and she is discharged to a
rehabilitation unit Six weeks after her original operation she is readmitted. She is drowsy,
hypotensive and bradycardic. An ECG shows low voltage complexes and a prolonged QT
interval Under-activity of which of the following glands gives the best explanation of the
clinical picture?
A. Adrenal
B Pancreas
C Parathyroid
D Pituitary
E Thyroid

98. A 36-year-old man falls on his outstretched right hand. Examination reveals
tenderness in the anatomical snuff box. Which one of the following tendons form a
boundary of the anatomical snuff box?
A Abductor pollicis brevis
B Extensor carpi radialis brevis
C Extensor carpi radialis longus
D Extensor indicis
E Extensor pollicis longus

99. A 65-year-old woman with metastatic breast cancer is admitted to hospital confused
and acutely unwell, with nausea and vomiting. Her vital signs are within normal limits.
What abnormality is most likely to contribute to her clinical condition?

A Hypercalcaemia
B Hyperkalemia
C Hypocalcaemia
D Hypokalemia .
E Hyponatremia'

100. A 4-year-old boy presents to the Emergency Department with a two-day history of
fever, difficulty walking and is unable to weight bear on the right leg. He has been on oral
Amoxycillin 250 mg three times a day for a chest infection over the last five days. He is
irritable with a temperature of 39.4°C. He does not allow examination and keeps his right
hip flexed and abducted. Blood tests are: white blood cell count; (WBC) 12.3 x -109/dL,
C-Reactive protein (CRP) 146, haemoglobin 11.3 g/dL What is the most likely diagnosis?

A. Acute avascular necrosis of hip


B. Dislocation of hip
C. Perthes disease
D Septic arthritis
E. Slipped upper femoral epiphysis
101. A 35-year-old woman presents with recurrent peptic ulceration. She is on proton
pump inhibitors and previously received Helicobacter pylori eradication therapy three
months ago. Which of the following is likely to be raised on venous blood testing?

A Cholecystokinin
B Gastrin
C Histamine
D Pancreozymin
E Secretin

102. A 55-year-old man presents with back ache. Neurological examination reveals lack of
extension of the left great toe.; Which of the following spinal cord segments are likely to
be involved?
A L1, L2 & L3
B L2, L3 & L4
C L4, L5 & S1
D S1, S2, & S3
E S3, S4 & S5

103. A 34-year-old man is seen in the Emergency Department with a fibular fracture
following football match. On examination he is noted to have loss of foot eversion. Which
area of skin should be examined to confirm. loss of the cutaneous distribution of the
affected-nerve?

A. Along the lateral aspect of the foot


B. Along the medial aspect of the foot
C. Cross the dorsal surface of the foot
D. On the plantar Surface of the foot
E. Over the heel

104. A 68-year-old man with insulin-dependent diabetes presents to the preadmission


clinic prior to surgery for rectal carcinoma. His creatinine is found to be 590 pmol/L. He is
referred to a renal unit where a measure of his glomerular filtration rate (GFR) is made.
Which of the following best describes why inulin could be used to measure his GFR?

A It is filtered and not reabsorbed


B It is filtered and reabsorbed
C It is filtered, secreted and reabsorbed'
D It is filtered, secreted but not reabsorbed
E It is not filtered but is secreted and not reabsorbed

105. A 60-year-old man presents with a one-month history of headache, confusion and
recent onset of right sided hemiparesis. A CT scan demonstrates a 4 cm >- 4 cm lesion with
central necrosis in the left frontal lobe. It is seen extending across the midline along the
corpus callosum. There is extensive oedema around the lesion. Which of the following is
the most likely diagnosis of this lesion?
A Central neurocytoma
B. Ependymoma
C. Glioblastoma
D. Meningioma
E. Oligodendroglioma

106. A 65-year-old woman presents with a 2.5 cm diameter mass in the upper outer
quadrant of the left breast with associated axillary lymphadenopathy. A core biopsy is
taken which confirms the presence of carcinoma. Which of the following types of
carcinoma is this most likely to be?
A Invasive ductal carcinoma
B Invasive lobular carcinoma
C Medullary carcinoma
D Mucinous carcinoma
E Tubular carcinoma

107. A 26-year-old man is admitted to the Emergency Department with multiple peripheral
fractures. He is clinically shocked. Which is the structure responsible for the first;
hemostatic response to a fall in systemic arterial blood pressure?

A. Adenohypophysis
B. Baroreceptor
C. Chemoreceptor
D. Kidney
E. Neurohypophysis

108. Vitamin K is required for normal blood clotting. Which one of the following
statements is true about the effects and availability of vitamin K?

A Affects platelet function


B Causes bleeding if taken in excess
C Is provided only by fresh food
D Is depleted by broad spectrum oral antibiotics
E Vitamin K absorption is affected by resection of the terminal ileum

109. In septic shock, norepinephrine (noradrenaline) is used to increase the systemic


vascular resistance. This action is a result of the stimulation of which of the following
receptors?
A α1
B α2
C β1
D β2
E. ∂1
110. A 10-year-old child presents with progressive facial weakness and a squint. On
examination there is a lower motor neuron facial weakness and failure of abduction of one
eye. An MRI scan shows a small, intrinsic mass lesion within the central nervous system.
Where is this most likely to be located?
A The cerebellar vermis
B The cerebellar tonsil
C The medulla
D The midbrain
E The pons
111. A 65-year-old man, with a history of ischaemic heart disease, has an abdomino-
perineal excision of the rectum after which he is stable. Two days later he is clammy, has
cool peripheries, a tachycardia and a blood pressure of 80/60 mmHg. His temperature is
normal. What is the most likely cause?

A. Anaphylactic shock
B. Cardiogenic shock
C. Hypovolaemic shock
D. Neurogenic shock
E. Septic shock

112. A 55-year-old1 man presents to his General Practitioner with a four-month history of
feeling tired. The General Practitioner notes breast enlargement. Investigations reveal:
Result Normal
TSH 0.2 mIU/L 0.4-4.3
Free T4 6 mmol/L 9-23
Which of the following is the most likely cause?
A Primary hyperthyroidism
B Primary hypothyroidism
C Secondary hyperthyroidism
D Secondary hypothyroidism
E Sick euthyroid syndrome

113. A 40-year-old woman presents with a parotid tumour. A biopsy reveals extensive
perineural invasion. Which is the most likely pathology?

A. Acinic cell carcinoma


B. Adenocarcinoma
C Adenoid cystic carcinoma
D. Lymphoma
E. Pleomorphic salivary adenoma

114. A 50-year-old man presents with malaise, abdominal pain, weight loss, fever and
myalgia Polyarteritis nodosa is associated with all of the following except:

A. Erythema nodosum
B. livedo reticularis
C mononeuritis multiplex
D nailfold infarcts
E palpable purpura.

115. A 30-year-old woman undergoes subtotal thyroidectomy. Five days later the wound
appears red and inflamed. Which of the following is the most likely causative organism?

A. Haemophilus influenzae
B. Proteus-mirabilis
C. Pseudomonas aeruginosa
D. Staphylococcus aureus
E. Streptococcus pyogenes

116. A 50-year-old man complains of recent onset diplopia. On examination he is unable to


lookinwards and downwards with his right eye; The most likely diagnosis is a lesion of the?

A. abducens nerve
B. ciliary ganglion
C. nasociliary nerve
D. oculomotor nerve
E. trochlear nerve

117. A 57-year-old heart transplant recipient is keen-to join the cardiac, rehabilitation
programme Which of the following factors is most likely to increase cardiac output in this
patient during moderate exercise?

A. Decreased negative intrathoracic pressure


B. Decreased venous tone
C Decreased ventricular compliance
D Increased atrial filling ITS
E. Increased intrapericardial Pressure

118. A 69-year man with recta! cancer undergoes low anterior resection. The liver is
disease free. The pathology of his tumour was reported as a large tumour invading from
the rectum into the mesorectum. Two nodes of 24 lymph nodes, close to the tumour were
carcinoma on histology. The tumour was completely excised. What is the correct
pathological staging of his tumour?

A Dukes' A
B Dukes' B
C Dukes' C
D Dukes' D
E T2 N1
119. A 21 -year-old man is admitted to the Emergency Department with a stab injury to his
right chest. Pulse rate is 110 beats/minute and blood pressure is 85/40 mmHg. Chest X-ray
shows a large right haemothorax. and a very small light apical pneumothorax. Which is the
first substance secreted in the process leading to increased renal reabsorption of sodium in
response to the above injury?

A. Angiotensin I
B. Angiotensin 11
C. Angiotensinogen
D. Antidiuretic hormone.
E. Renin

120. During the second (proliferative) phase of wound healing the predominant cells in the
wound site are fibroblasts. This cell is of mesenchymal origin and produces the matrix and
collagen needed to strengthen the scar. Cross linkage of collagen requires hydroxy-proline
and hydroxy-lysine residues, which requires a specific vitamin to be available in sufficient
quantities. Deficiency of which vitamin results in collagen that is unstable?

A Vitamin B2 (riboflavine)
B Vitamin B6 (pyridoxine)
C Vitamin C (ascorbic acid)
D Vitamin D (cholecalciferol)
E Vitamin E (tocopherol)

121. A 62-year-old woman presents with a firm irregular mass in the upper outer quadrant
of the right breast. This appears malignant on mammography, and fine needle aspiration
cytology is reported as Co. Which is the first lymph node to which the tumour is most likely
to metastasise?

A Initial node
B Primary riode
C Secondary node
D Sentinel node
E Virchow's node

122. Which of the following statements is true with relation to cardiac muscle?

A Contraction does not involve release of Ca by the sarcoplasmic reticulum


B. Contraction is triggered by an influx of Ca through the sarcolemma
C. Excitation is passed from one cardiac cell to another through desmosomes
D. Hypertrophy is brought about by division of existing cardiac cells
E. There is capacity for regeneration

123. A 70-year-old retired farm worker presents with a scaly lesion on the back of his left
wrist. Biopsy shows enlarged pleomorphic squamous cells with mitoses that have
breached the basement membrane. What is the name of this process?
A. Dysplasia
B. Hyperplasia
C. Hypertrophy
D Metaplasia
E Neoplasia

124. A 45-year-old man has established cirrhosis. At a follow-up appointment a palpable


spleen four fingers breadths below the costal margin, is noticed. Full blood counts have
shown a persistent thrombocytopaenia. Bone marrow examination shows megakaryocyte
hyperplasia. What is the most likely cause of the thrombocytopaenia?

A. Ineffective production in the bone marrow


B. Platelet destruction in the bone marrow
C. Platelet destruction in the liver
D Platelet destruction in the spleen
E Platelet storage in the spleen

125. A surgeon is carrying out an elective splenectomy for congenital spherocytosis. Which
structure will be divided in order to mobilize the spleen from the posterior abdominal wall?

A Gastrosplenic ligament
B Lesser omentum
C Lienorenal ligament
D Phrenicocolic ligament
E. Short gastric vessels

126. A 50-year-old man is admitted with epigastric pain and a rigid abdomen He is
diagnosed with, a perforated duodenal ulcer. Ulceration due to excessive gastrin activity
may be: caused by increased levels of?

A. adrenaline (epinephrine)
B. calcitonin
C glucagon
D secretin
E. somatostatin

127. A 66-year-old woman with known tumour of her spine has started to retain urine and
is experiencing decreased anal and rectal tone. The tumour is pressing directly on the
conus medullaris. At which one of the following vertebral levels is the tumour most likely
to be located?

A. T9/T10
B T11/T12
C L1/L2
D L3/C4
E L5/S1
128. A 26-year-old man presents with a two-month history of unilateral testicular swelling.
An ultrasound scan shows a heterogeneous mass within the testis with surrounding fluid.
His blood test show- an elevated alpha fetoprotein level. Which of the following is the
most likely diagnosis?

A Lymphoma
B Orchitis
C Seminoma
D Teratoma
E Tuberculosis

129. A 50-year-old woman presents with a history of faecal incontinence over the past few
year She had a prolonged and difficult first stage of labor 20 years previously. Physical
examination reveals a relatively lax anal sphincter. Which nerve is likely to have been
damaged in labour?

A Autonomic nerves to the rectum


B Genitofemoral nerve
C Lumbosacral trunk
D Obturator nerve
E Pudendal nerve

130. A 70-year-old man is admitted to hospital with a 12-hour history of a painful white leg.
Afemoral artery embolus is shown on angiography. He is an insulin-controlled diabetic and
takes ibuprofen for longstanding osteoarthritis. Twenty-four hours after emergency
embolectomy his blood pressure is 90/60 mmHg and he passes very dark brown urine.
A dipstick shows myoglobin and traces of blood. His blood glucose is 15 mmol/L (normal-
4.0-6.0), urea 12 mmol/L (normal 3.2-7.5) and creatinine 180 mmol/L (normal 35-110). What
is the most likely cause of his abnormal renal function?

A Drug induced nephropathy


B Hypovoiaemia
C Methaemoglobinuria
D Rhabdornyolysis
E. Uncontrolled diabetes

131. An 86-year-old woman, with a history of partial gastrectomy, is in the high


dependency unit for a chest infection that is being treated with antibiotics. She complains
of difficulty swallowing. Endoscopy shows multiple white plaques on the mucosal surface
of the oesophagus. Biopsy is most likely to show:
A. acid reflux oesophagitis
B Barrett's oesophagitis
C candida oesophagitis :
D eosinophilic oesophagitis
E. herpes oesophagitis
132. A 65-year-old woman has been given a total of 15 mg of morphine/over a 30-minute
period for pain relief following a femoral neck fracture, mellowing observations are found:
Sp02 on 80% oxygen is 80%; blood pressure 90/60 respiratory rate 6 breaths/minute.-The
immediate intervention should be:

A. high flow oxygen


B, intravenous fluid replacement
C intravenous Naloxone
D. monitor her ECG
E. tracheal intubation

133. The following drugs are used to treat patients with peripheral vascular disease. They
all reduce the rate of myocardial infarction and stroke except:
A. A.C.E inhibitors
B. antihypertensive medication
C. aspirin
D. cholesterol lowering drugs
E. oxypentifylline

134. A 4-year-old boy, is brought to the Emergency Department with a painful right elbow.
His father stales that he was swinging his son by his arms when the pain came on suddenly
An X-ray shows that the radial head is displaced from its usual position. What is the
ligament (of the proximal radio-ulnar joint) that holds the radial head in place?

A Annular ligament
B Conoid ligament
C Quadrate ligament
D Radial ligament
E Ulnar ligament

135. A 35-year-old man presents with a three-week history of low back pain and a three-
day history of pain and weakness in his left leg. Physical examination demonstrates
numbness over the posterior aspect of the left calf extending to the lateral aspect of the
foot. Left ankle reflex is absent. A magnetic resonance scan shows compression of the left
S1 nerve root. What is the most likely responsible structure?

A. Annulus fibrosus
B. Anterior longitudinal ligament
C. Nucleus pulposus
D. posterior longitudinal ligament
E. Vertebral end plate

End of paper 1
1. A 26 years old man motorcyclist loses 1 L of blood secondary to an open fracture of the
femur sustained in RTA. Which of the following is the most likely earliest compensatory
response in hypovolemeia in the this patient?

A. Baroreceptors induced venoconstriction


B Capillary fluid shifts
C Decreased atrial natriuretic peptide (AMP),secretion
D Renal fluid retention
E Renin Angiotensin system activation

2. Histology of a discrete palpable lump in the breast o 1 a 34-year-oM woman has shown
Apocrine metaplasia, epithelial overgrowth and papillary projections. What is the most
Likely pathological process ?

A Benign breast cyst


B Carcinoma of the breast
C Fibroadenoma
D Phylloids tumor
E Plasma mastiditis

3. A 35 -year-old woman 'presents with recurrent peptic ulceration. She is on a proton


pump inhibitor previously received Helicobacter pylori eradication therapy three
months ago. Which of the following is likely to be raised on venous blood testing?

A Cholecystokinin
B Gastrin
C Histamine
D Pancreozymin
R Secretin

4. 3 Year old male boy presented with his mother for rectal blood loss, his mother
describes it as a bright red blood in the toilet pan. He has no pain on defecation with
negative family history. What is the most likely cause?

A-Adenomatous polyp
B Juvenile polyp
C Metastatic polyp
D Multiple polyposis coli
E peutz jegher syndrome

5. A 32-years old man presents with a painful torticollis there is no past medical history and
his only complaint is that he has been feeling rather tired over the last 4 nights. On
examination, he has large rubbery mass in the lateral aspect of his neck as well as a few
smaller masses along his internal jugular vein. You correctly assume that the muscular
neck spasm and the large mass are connected. The most likely cause of his torticollis is due
to pressure on?
A. Ansa cervicalis
B. Cervical plexus branch
C. Cranial accessory nerve
D. Spinal accessory nerve
E. Vagus nerve

6. A thin menopausal 52-vear-frld woman is diagnosed with breast cancer. Her menarche
was at the age of 14. she had her first child at age40 years. During lactation, she
developed breast abscess that necessitated surgical intervention. She has no family
history of breast cancer. Which of the following is the most significant risk factor for
this patient?

A Age at first full term pregnancy


B Familial predisposition
C History of breast abscess
D Interval between in menarche and menopause
E Physical stature

7. A 30-ycar-old woman is sent to the outpatient clinic with weight loss of 5 kg over the
Last 6 months, She also complains of anxiety, panic attacks and palpations. On
Examination. There is a swelling in the anterior neck which moves on swallowing
Which of the following is most likely pathology underlying this presentation?
A Graves disease
B Hasliimoto's thyroiditis
C Metastasis to the thyroid
D. Papillary carcinoma
E Undifferentiated carcinoma

8. A 69-yeac-oJd man has been admitted to the high dependency unit following an
anterior resection under general anesthesia. He was given 2 mg of intrathecal
morphine. On examination, he looks pale and drowsy. Arterial blood gasses results are:

PH pCo2 pO2 Base Gluose Lactate


Excess
Finding 7.28 8.1 kPA 10.2 kPa -2.1 21 mmol/L 4.0
mmol/L
Normal 7.35-7.45 4.7-6.0 11.9-13.3 -2 to +2 4-7 0.5-2.2

Which o f the following is the most likely diagnosis?


A. Diabetic ketoacidosis
B. Metabolic acidosis
C. Metabolic alkalosis
D Respiratory acidosis
E Respiratory Alkalosis
9. A 32 year-old woman has a pigmented lesion excised from her left calf. The
histopathological diagnosis is melanoma in situ, which is completely excised with a-
1 cm margin. What is the next most appropriate management?

A Education about skin self-examination and discharge from follow up


B. Elective inguinal lymph node dissection
C Re excision with 2 cm margin
D. Removal of any other pigmented lesions
E. Sentinel lymph node biopsy

10. A 60-year-old man with a past history of angina undergoes an uncomplicated


operation, for an inguinal hernia. Postoperatively he is found to be hypotensive,
tachycardic, and has a raised jugular venous pressure. What is the most likely Underlying
cause Of his hypotension?

A. Reduced afterload
B. Reduced parasympathetic tone
G Reduced preload
D. Reduced stroke volume
E. Reduced sympathetic tone

11. A 4-years old boy presents to the Emergency Department with a two-day history of
fever, difficulty walking and is unable to weight bear on the right leg. He has been
on oral Amoxycillin 250 mg three times a day for a chest infection over the last five
days. He is irritable with a temperature of 39.4°C. He does not allow examination
and keeps his right hip flexed and adducted. Blood tests are, (WBC) 18.3 (CRP) 146,
haemoglobin 11.3 g/dL. What is the most likely diagnosis?

A Acute avascular necrosis of hip


B Dislocation of hip
C Perthes disease
D Septic arthritis
E. Slipped upper femoral epiphysis

12. A 34-year-oid man is admitted to-the Emergency Department with a head injury. On
examination his Glasgow coma scale is 9. A CT scan of the brain demonstrates an extra
dural haemorrhage. Which of the following arteries is the most likely source of this?

A. Anterior cerebral
B. Middle cerebral
C. Middle meningeal
D Posterior cerebral
H. Superficial temporal

13. A 65-year-old man has a history of transient ischemic attacks. He is due to undergo
carotid endarterectomy. Which one of the following is true of the internal carotid artery?
A. Begins at the level of the sixth cervical vertebra
B. Divides into the anterior, middle and posterior cerebral arteries
C. Gives off the ophthalmic artery
D. Is accompanied within the skull by preganglionic sympathetic nerves
E. Passes through the foramen ovale

14. A 25-year-old male athlete is in training. At rest, how many liters of blood per
minute does his heart pump out?
A. 0.9
B. 2 to 3
C 5 to 6
D 8 to 10
E 15 to 20

15. A 26-year-oJd man presents to the Emergency Department after sustaining a glass
cut to his arm. On examination there is a 10 cm longitudinal laceration on the
anterior aspect of his upper arm. He has symptoms suggestive of ulnar nerve injury.
On exploring the upper part of the arm we would expect the ulnar nerve to be:

A. anterior then medial to the brachial artery


B. medial then anterior to the brachial artery
C medial to the brachial artery
D. posterior then medial to the brachial artery
E. posterior to the brachial artery
16. A 55-year-old man presents with acute back pain following a severe road traffic
accident. Neurological examination reveals lack of sensation of the umbilicus and
below. What is the spinal level of the neurological deficit likely to be?

A T8
B T9
C T10
D T11
E T12
17. A- 65-year-oid man complains of being thirsty and getting up in the middle of the
night to get to the toilet. His weight is 95.5 kg. height 1.65 in and blood pressure
167/94 mmHg. An oral glucose tolerance test was performed and produced the
following results:
The most likely diagnosis: Fasting plasma glucose 5.9 mmol/L
A Diabetes insipidus Two hours plasma 11.3 mmol/L
B Diabetes mellitus glucose
C Impaired fasting Glycemia
D impaired glucose tolerance
E Normoglycemia
18. in a 9-year-old child with cellulitis of the hand, which chronological sequence of
immunoglobulin production is correct?

A. IgD precedes IgE production


B IgG precedes IgA production
C IgG precedes IgE production
D IgM precedes IgA production
E IgM precedes IgG production

19. A 60-year»old man presents to the-Emergency Department with epistaxis. The source
0f the bleeding is identified as Little's area and resolves with direct cautery. Which vessel is
most likely responsible for the bleeding?

A. Anterior ethmoidal artery


B. Infraorbital artery
C. Middle meningeal artery
D Sphenopalatine artery
E. Supratrochlear artery

20. A 75-year-old man presents with esophageal reflux. Endoscopy confirms the
Presence of a hiatus hernia. The esophagus passes through the diaphragm at which
level?

A T8
B T9
C T10
D T11
E T12

21. A 78-year-ojd woman with emphysema receiving 28% oxygen by mask has the
Following blood gas results
PH pO2 pCo2 Bicarbonate Base Excess
Finding 7.28 6.2 kPa 8 kPA 36 mmol/L +5
Normal 7.35-7.45 11.9-13.3 4.7-6.0 22-26 -2 to +2
Which of the following is the most likely interpretation?

A Mixed respiratory and metabolic Acidosis


B Partially compensated metabolic acidosis
C Partially compensated respiratory acidosis
D Uncompensated metabolic acidosis
E Uncompensated respiratory acidosis
22, A 26-year-old man is having a stereotactic frame fitted to his skull prior to
radiosurgery on a cerebral arteriovenous malformation. Four pins secure the frame
tightly through the scalp to the outer table of the skull, two anteriorly, two
posteriorly. On insertion of one of the-posterior pins, arterial haemorrhage is encountered.
Which artery is most likely to have been punctured?

A The ascending pharyngeal artery


B The middle meningeal artery
C The occipital artery
D The posterior cerebral artery
E The posterior communicating artery

23. A. 2.6-year-old man presents with a two-month history of unilateral testicular


swelling. An ultrasound scan shows a heterogeneous mass within the testis with
surrounding fluid. His blood test shows an elevated alpha fetoprotein level Which of
the following is the most likely diagnosis?

A Lymphoma
B Orchitis
C Seminoma
D Teratoma
E Tuberculosis

24. A 2 day old baby presents with increasing respiratory distress. He was born at term
By normal vaginal delivery. On examination he has cyanosis of the lower limbs and marked
respiratory in drawing of the chest His femoral pulses ate absent bilaterally and he has been anuric
for the last 2 hours. Pulse rate is 140 beats/minute, regular, and his blood pressure is 60/30 mmhg
in both upper limbs. What is the most likely diagnosis?

A. Hypoplastic left heart syndrome


B. Interrupted aortic arch
C. Pulmonary atresia and ventricular septal defect
D. Transposition of the great arteries
E. Tricuspid atresia

25. A 50-years old woman presents with a history of right upper quadrant pain and
jaundice. She reports that her urine was dark in colour and that her stools are
offensive and difficult to flush. Which of the following explains the dark urine?

A. Increase in conjugated bilirubinuria


B. Increase in unconjugated bilirubinuria
C. increase in urea excretion
D Increase in urinary urobilinogen
H. Reduced enterohepatic bile salt circulation
26. An 80-year-old woman, who has suffered a fall, is found lying on the floor, where,
she has-been for over 12 hours. Initial assessment shows that she has a core
temperature of 28°C. What would the expected early physiological response to her
body temperature be?

A. Increased fat metabolism


B. Increased constriction of peripheral blood vessels
C. Increased hypothalamic set-point
D. Increased metabolic rate.
E. Increased thyroid activity

27. A -12-year-old boy presents to the Emergency Department two hours after helping
his father cut the grass. He complains of rhinorrhoea, itchy eyes, sneezing and a
a blocked nose. He is apyrexial with a haemoglobin of 12.2 g/dL and white blood cell
count of 6.8 * I09/L with a raised eosinophil count. Chest X-ray is; clear. Which
immunoglobulin is most likely to cause this reaction?

A. IgA
B. JgD
C. IgE
D. IgG
E. IgM

28. A 45-year-old man presented with backache and leg pain due to –prolapsed lumbar •
intervertebral' disc. The pain, which is aggravated by coughing and sneezing; radiates
to the dorsum of the foot On examination there is weakness, of the dorsiflexion of
the foot. Which nerve root is most likely to be involved?

A. T12
8- L3
C. L5
D. S1
E. S2

29. A 75-year-old woman, who has a carcinoma of the upper rectum, undergoes
anterior resection. The arterial blood supply of the upper rectum arises from which
of the following?

A. Coeliac artery
B. Iliocolic artery
C. Inferior mesenteric artery
D. Internal iliac artery
E. Superior mesenteric artery

30. A 65-year-old man is undergoing an abdominal aortogram, A stenosis is


demonstrated in a lateral aortic branch, arising at the level of the body of the second
lumbar vertebra. The stenosed vessel is most likely to be the:

A. coeliac artery
B. inferior mesenteric artery
C. left renal artery
D. second left lumbar artery
E. superior mesenteric artery

31. A 40-year-old man is admitted tc the surgical day case unit for repair of his left
inguinal hernia. On examination he is noted to have diffuse skin tanning, spotty
pigmentation of the elbows, nipples and buttocks, and pigmentation of the scar from
A previous right inguinal hernia repair. Three hours after the operation be becomes
severely hypotensive. 'What is the most likely cause?

A. ACTH deficiency
B. Adrenal insufficiency
C. Growth hormone deficiency
D. Potassium deficiency
E. Thyroxine deficiency

32. A 19-year-old woman presents to the Emergency .Department profoundly


hypovolemic having fallen from a horse. A postero-anterior (PA) chest radiograph
shows a fracture to the medial third of the left clavicle. Which of the following
vessels was-most likely damaged?

A Brachiocephalic trunk
B Left axillary artery
C Left common carotid artery
D Left subclavian artery
E Left vertebral artery

33. 65 years old man presented with an inguino-scrotal swelling in the right groin which is
non tender. A cough impulses is elicited, At operation, an indirect inguinal hernia is
repaired. The cremasteric muscle is derived from which of the following

A. External oblique aponeurosis


B Internal oblique muscle •
C. Rectus abdaminis muscle
D. Rectus sheath
E. Trsusversalis fascia
34. A 70-y ear-old woman is in the recovery area and receives 28% oxygen by mask.
Blood gas shows:
PH Pco2 Po2
Findings 7.1 10.0 kPa 12 kPa
Normal 7.35-7.45 4.7-6.0 11.9-13.3
Reduced sensitivity of which receptors is most likely to be responsible for this blood gasses
picture?
A. Adrenergic receptors
B. Baro receptors
C Central chemo receptors
D. J receptors
E Lung stretch-receptors

35. A 28-year-old man presents with pain in his left scrotum. A diagnosis of varicocele
is made. Which vessel is involved?

A Femoral vein
E. Inferior epigastric vein
C. Long saphenous vein
D Internal pudendal vein
E Testicular vein

36. A 60-year-old man, who is a heavy smoker, presents with a 10-day history of frank
painless haematuria. His prostate is slightly enlarged on rectal examination. his
haemoglobin is 11.3 g/dL. creatinine 84 pmol/L and prostate specific antigen (PSA.)
is 3,1 mg/dL. What is the most likely pathological process?

A. Benign prostatic hyperplasia


B. Prostate cancer
C. Renal cell carcinoma
D. Transitional cell carcinoma of bladder
E. Urinary tract infection

37-. A 34-year-old woman presents with an irregular mass in the right breast which is
clinically. radiologically and histologically malignant. Her mother died of breast
cancer at the age of 58 and her grandmother died of ovarian- cancer at the age of 55.
Which gene is most likely to be involved in the development of this woman tumor?
A BRCA1
B. hMLH1
C. KRAS
D. p53
E. PTEN

38. 65-year-old man presents with a non-tender swelling in the right hemi-scrotum. At
operation, the hydrocele sac is opened and 400 ml of fluid is drained. Which
anatomical structure surrounds the fluid?

A. Dartos muscle
B. Patent processus vaginalis
C. Testicular capsule
D. Tunica albuginea
E. Tunica vaginalis

39. A 24 years old man is brought to the Emergency Department following a road traffic
accident. He has obvious airway compromise due to mouth/ facial haemorrhage
which requires a surgical airway. The surface landmarks used to localise, the optimal
site for a tracheostomy incision are midway between the suprasternal notch and which of
the following?

A Cricoid
B Hyoid
C Mastoid
D Laryngeal prominence
E Thyroid

40. A 60-year-old man has -an anterior resection for a high rectal carcinoma. The
histopathology report indicates the lesion is Dukes' stage B. What is the approximate
average five years survival rate for patients with these lesions?

A. 10%
B. 20%
C. 35%
D 70%
E 90%

41. A 20-years man presents to the Emergency Department after accidentally tripping
over and lacerating his hand on a glass bottle. On examination there is a 2 cm
laceration on the hypothenar eminence with loss of flexion in the distal
interphalangeal joint of the little finger. What is the most likely tendon to be injured?

A Flexor digiti minimi brevis


B Flexor digitorum profundus
C Flexor digitorum superficialis
D Fourth palmar interosseous
E Lumbirical

42 A 75-year-old man, who smokes heavily, underwent coronary artery bypass grafting
six months ago. A left internal-mammary artery graft was used. He now complains
of angina on pegging his laundry on the washing line to dry. Which of the following
lesions best explains his symptoms?

A. .Micro-emboli from the left common carotid artery


B. Micro-emboli from the le\t internal carotid artery
C. Stenosis.of the subclavian artery at the level of the costo-cervical trunk
D. Stenosis of the subclavian artery distal to the insertion of scalenus anterior
E. Stenosis of the subclavian artery proximal to the first branch
45. A 50-year-old alcoholic man presents-to the Emergency Department and is found to
be unable to extend his wrist, thumb and fingers of his right hand. He is also found
to have weak extension of the right elbow joint and loss of sensation on the dorsum
of the first web space . Which nerve is most likely to have been injured?

A Median nerve
B Musculocutaneous nerve
C Posterior interosseous nerve
D Radial nerve
B. Ulnar nerve

44. A 55 year-old man presents with acute back pain following a severe road traffic
accident. Neurological examination reveals lack of dorsi-flexion of the left ankle
joint. Which of the following spinal cord, segments are most likely to be injured?

A. L1, L2 & L3
B. L2, L3 & L4
C L4, L5 & S1
D. S1, S2& S3
E. S3, S4 & S5

45. A 21 -year-old man is admitted to the Emergency Department with a stab injury to
his right chest. Pulse rate is 110 beats/minute and blood pressure is 85/40 mmhg
Chest X-ray shows a large right haemothorax and a very small right apical
pneumothorax. Which is the first substance secreted in the process leading to increased
renal reabsorption of sodium in response to the above injury?

A. Angiotensin I
B Angiotensin II
C Angiotensinogen
D Antidiuretic hormone
E Renin

46. A 21-year-old man has been stabbed in the back of the knee, dividing the popliteal
artery, and is undergoing repair via a posterior approach. Which of the following
structures is most likely to be encountered first, when dissecting deep from the skin „
incision?

A Popliteal artery
B Popliteus muscle
C Popliteal vein
D Tibial nerve
E Soleus muscle
47. A 78-year-old woman presents with urinary urgency and incontinence. The external
urethral sphincter is innervated by which of the following nerve roots?

A. L 3, 4 & 5
B. L4, 5 & SI
C. L 5, S1 & 2
D. S1, 2 & 3
E. S2.3& 4

48. A 40-.year-Qld woman presents with a parotid tumour. A biopsy reveals perineural
invasion. Which is the most likely pathology?

A. Acinic cell .carcinoma


B . Adenocarcinoma
C. Adenoid cystic carcinoma
D. Lymphoma
E. Pleomorphic salivary adenoma

49. A 19-year-old man was assaulted and sustained injuries to the right side of his head.
After two weeks he notices dial his right eye is dry and it could not produce tears.
from which ganglion is post synaptic fibers arise to supply lacrimal gland?

A. Geniculate ganglion
B. Inferior ganglion of the vagus nerve
C. Otic ganglion
D. Pterygopalatine ganglion
E. Superior cervical ganglion

50. A 35-year-old man undergoes a right inguinal hernia repair under genera! anaesthetic
as a day patient. He has a nerve block after the procedure. On recovery he has
weakness of the right leg. Which nerve has been affected?

A. Femoral nerve
B. Genitofemoral nerve
C Ilioinguinal nerve
D Lateral cutaneous nerve
E Sciatic nerve
51. A man suffers a brachial plexus injury. On examination, he has a Horner's syndrome
in association with upper limb paralysis. Which nerve roots does the Homer's
syndrome suggest involvement of?

A. C2 & C3 roots
B. C4 & C5 roots
C. C6 & C7 roots
D C8 & T1 roots
E. T2 & T3 roots
52. A 60-year-old woman with breast carcinoma complains of difficulty chewing her. .
food and is found to have numbness of the lower lip on one side. CT scanning shows
a small metastatic lesion affecting the bony skull base on. The same side as the lip
numbness. Which foramen is the most likely to involve?

A. Foramen caecum
B Foramen magnum
C Foramen ovale
D. Foramen rotundum
E. Foramen spinosum

53. Which one of the following muscles is an extensor of the hip?

A. Adductor longus
B. Gracilis
C. Iliopsoas
D. Pectineus
E. Semitendinosus

54. In order to expose the right axillary artery, a transverse skin incision is typically
made below the clavicle from a point just lateral- to the sternal end of the clavicle to
the deltopectoral groove. Which of the following structures would be encountered in
the dissection down to the vessel?

A. Lateral thoracic artery


B. Phrenic nerve
C. Suprascapular artery
D. Thoracic duct
E. Thoraco-acromial artery

55. A 55-year-bld man presents to the-Emergency Department after collapsing. On


examination his pulse is 124 beats/minute, blood pressure is 60/30 mmHg,
respiratory rate is 34 breaths/minute and his peripheries are warm. What is the most
likely diagnosis, for this patient?

A. Cardiac failure
B. Haemorrhage
C Hypovolemia
D. Pulmonary embolus
E. Sepsis

56. In the posterolateral approach to a posterior malleolar fracture, an incision is made


between the calcaneal (Achilles) tendon and the distal-.fibula. Which of the following
structures is at risk?

A. Deep peroneal (fibular) nerve


B. Saphenous nerve
C. Superficial peroneal, (fibular) nerve
D Sural nerve
E. Tibial nerve

57. A 25-year-old woman undergoes.an elective right thoracoscopic procedure for


treatment of right palmar hyperhidrosis. Diathermy is applied to a neural structure
lying anterior to-the neck of tire right first rib. What is the most likely complication
to occur as a result of this procedure?

A. Bradycardia
B. Hoarseness
C. Homer's syndrome
D. Raised right hemi diaphragm
E. Reduced right biceps tendon reflex

58 / A 33-year-old man presents to the orthopedic outpatient clinic with a six-month


history of low back pain radiating to the lateral aspect of the left upper thigh. Micturition
and defecation are normal and there is no history of previous Injury. On examination, left
lateral flexion of the spine is limited but a full range of hip movements are observed,
although it is painful. Sensation is altered over the front of the knee. Left knee reflex is
reduced. Which spinal nerve-is most likely to have been compressed?

A. L1
B. L2
C. L3
D. L4
E. L5

59. A 2S-year old motorcyclist is admitted following a road traffic accident, having
sustained- bilateral femoral fractures and a ruptured spleen. Three days postoperatively lie
is noted to be confused, hypoxemic and difficult to ventilate. His observations show a
blood pressure, of 120/8O.mmHg, regular pulse of 88 beats/minute and he is apyrexial. A
chest X-ray shows bilateral diffuse lung infiltrates. What is the most likely underlying
diagnosis?
A. Adult respiratory distress syndrome
B Atelectasis
C Bronchopneumonia
D. Pulmonary oedema
E: Pulmonary thrombo-emboilism

60. On-an ultrasound scan of the popliteal fossa when investigating-a swelling, which of
the following structures is closest to the capsule of the knee joint?
A Great (long) saphenous vein
B Popliteal artery
C Popliteal vein
D. Small (short) saphenous vein
E. Tibial nerve

61. A 25 years old man sustains a twisting injury while playing football. He develops
immediate swelling of the knee and he can not continue the game. Six months later,
he Is still not able to play football. His knee feels unsteady and tends to give way. On
examination, he has a full range of knee motion. There is a positive anterior draw
test and a small effusion. What is the most likely structure damaged?

A Anterior cruciate ligament


B. Lateral collateral ligament
C. Medial collateral ligament -
D. Oblique popliteal ligament
E. Posterior cruciate ligament

62. A 62-year-old man required a partial gastrectomy for a large benign ulcer in his gastric
antrum. When he is reviewed in clinic, six months later he complains of palpitations,
weakness and sweating along with a cramp-like abdominal pain which occurs within an
hour of eating n meal. Usually, he has to lie down for 30-40 minutes until the symptoms
subside. What is the most likely cause of his symptom pattern?

A Chronic gastroparesis
B Dumping syndrome
C. Delayed gastric emptying
D Reflux gastritis
E Recurrent ulceration
63. in septic shock, noradrenaline is used to increase the systemic vascular resistance This
action is a result of the stimulation of which of the following ?

A α1
B α2
C β1
D β2
E. ∂1

64. A 56-year-old man presents with acute epigastric pain and vomiting. On examination
he has guarding in the upper abdomen His test results are:
Value Normal
Serum amylase 900 IU/L ≤100
Serum ALT 61 IU/l ≤50
Alkaline Phosphatase 93 IU/l 20-120
Albumin 38 g/L 35-50
Gamma GT 500 IU/L ≤60
Bilirubin 15 umol/L 0-20
Triglyceride 2.9 mmol\L ≤1.7
Which of the following is the most likely etiology of this condition?
A. Alcohol
B. Choledocholithiasis
C. Hypertriglyceridemia
D. Hypocalcaemia
E. Mump

65. A 47-year-old woman with end stage renal failure requires an elective
cholecystectomy for symptomatic gallstone disease. Her hemoglobin preoperatively is 7.2
g/dL. What is the major cause of her anaemia?

A. Calcium deficiency
B. Erythropoietin deficiency
C Folate deficiency
D Iron deficiency
E. Vitamin B12 deficiency

66. A 60-year-old man undergoes cystectomy a bladder carcinoma. During surgery,


the ureters are identified. On which region of the bladder do the ureters pierce the
bladder wall?

A. Anterior surface
B. Apex
C. Lateral surfaces
D. Neck
E. Posterior surface

67. A 65 year-old man had a colonic resection for carcinoma 12 hours ago. He is now
passing concentrated urine at a rale of 0,5 ml/kg/hr. Which endocrine response is most
likely to have caused this?

A. Decreased aldosterone release


B. Decreased insulin release
C. Decreased thyroxine release
D Increased adreno-coriticotrophic hormone (ACTH) release
E Increased Vassopressin release

68. A 28-year-old man presents with an ischiorectal abscess. Where is ths abscess
cavity likely to be sited?

A. Above the levator ani


B. Between external and internal anal sphincter
C. Lateral to the obturator internus
D Medial to the internal anal-sphincter
E Medial to the pudendal canal
69. A 36-year-pld man falls on his outstretched right hand.-Examination reveals •
tenderness in the anatomical snuffbox. Which one of the following tendons form a
boundary of the anatomical snuff box?

A. Abductor pollicus brevis .


B Extensor carpi radialis brevis
C. Extensor cairpi radialis longus
D. Extensor indicis
E. Extensor poliicis longus

70. A 55-year-old- woman complains of cramps and tingling in her legs and arms 48
hours following a sub total thyroidectomy. Her vital signs, are stable. Investigations
reveals a normal white-cell count and normal haemoglobin. The NA is 132 mmol/L K+3.2
mmoI/L and Ca 1.60 mmol/L. Serum albumin is normal The next appropriate step is
intravenous infusion of?

A. Calcium chloride
B. Magnesium sulphate
C. Potassium chloride
D. Sodium bicarbonate
E. Sodium chloride

71. A 40-year-old. woman presents with faecal incontinence and; anatomically intact
internal and external anal- sphincters. Which structure is most likely to have been
damaged?

A. First sacral nerve-root


B. Obturator nerve:
C. Pelvic splanchnic nerves
D. perineal nerve
E. Pudendal nerve

72. Abdominal free fluid will collect in the lowest part of the peritoneal cavity. At
operation with the patient supine, in which of the following will the fluid collect first?

A Hepatorenal pouch
B. Left subphrenic space
C. Lesser sac
D. Right paracolic gutter
E. Right subphrenic space

73. A 45-year-old homeless man presents with a cough and weight loss over three
months. On examination his BMI (Body Mass Index) is 19 and he has reduced
breath sounds in the right upper zone. His chest X-ray shows a cavitaing lesion in
the right upper lobe. He undergoes a bronchoscopy and a brornchial biopsy. The
biopsy shows featureless necrosis surrounded by epithelioid macophages and giant
cells. Which of the following is the most likely diagnosis?

A.. Actinomycosis
B Bronchiectasis
C Sarcoidosis
D Squamous cell carcinoma
E. Tuberculosis
74. A healthy 36 years old man is being assessed with a view to being a life related kidney
donor which are of following investigations is most accurate for measuring the GFR?

A Creatinine clearance
B Glucose Clarence
C Inulin Clarence
D PAH
E Urea clearance

75. A 34-year-old pregnant woman develops a swollen leg. Her mother and maternal
aunt also had a similar problem during their pregnancies. Which of the following
tests is likely to be positive?

A. Antiendomysial antibodies
B. Antimitochondrial antibodies
C. Antinulear antibodies
D. Antiphospholipid antibodies
E. Antithyroglobulin antibodies

76. A 72-year-old alcoholic man is known to suffer from chronic obstructive pulmonary
disease (COPD). He has now been diagnosed with carcinoma of the urinary bladder
and-has a history of a gastric ulcer. Which of the following agents is most likely to
be responsible for all these problems?

A. Alcohol
B Cigarette smoking
C. Drugs
D. Dust
E. Virus

77. A 3-year-old boy is admitted to hospital with severe vomiting. Radiographic


examination and history reveals that he is suffering from annular pancreas. Which of
these structures is constricted?
A. First part of duodenum
B Second part of duodenum
C. Third part of duodenum
D. Proximal jejunum
E. Pylorus of stomach
78. A 45 year-old man presents with a 3 cm * 4 cm swelling in the right groin which is
non-tender. A cough impulse is elicited. At operation, an indirect inguinal hernia is
found. The external inguinal ring is a defect in which of the following abdominal

A. External oblique aponeurosis


B. External oblique muscle
C. Internal oblique muscle
D Transversus abdominis muscle
E Transversalis fascia

79. A 45-year-old woman suffers from spina bifida and is confined to a wheelchair. Her
legs are not fully developed. What is the pathological process which has occurred in
the legs?

A. Apoptosis
B Atrophy
C. Hyperplasia
D. Hypertrophy
E. Hypoplasia

80. A 23-year-old man presents to the Emergency Department-with non-specific: chest


pains. A postero-anterior (PA) chest radiograph is performed and is normal. Immediately
inferior cauidal to the outline of the aortic knuckle was a farther structure with a convex
border. What is this structure?

A. The left atrium


B. The pulmonary trunk
C. The oesophagus
D. The right atrium
E. The superior vena cava

8I. A 45-year-old woman presents to the hospital with jaundice. She- undergoes an
ultrasound scan and is diagnosed with a hepatocellular carcinoma affecting the left
lobe of her liver. Which virus is the most likely to be implicated?

A. Epstein Bar virus (EBV)


B. Hepatitis B virus (HBV)
C. HIV
D. Human papilloma virus (HPV)
E. Human T cell lymph a trophic virus-1 (HTLV1)

82. An 82-year-old man has complete occlusion of his inferior mesenteric artery on
angiography but no symptoms or .signs of colonic ischaemia. Which of the following
arteries is the .most likely additional source of blood supply to the territory of the
inferior mesenteric artery?
A Left colic artery
B Left gastroepiploic
C. Middle colic
D. Splenic
E. Superior rectal

83. A 56-year-old man with documented Type 1 diabetes mellitus presents to the
Surgical outpatient clinic with 3 months history of right leg pain brought on by
walking 50 meters. He has no history of back pain. He has been a smoker for 10 years (10
cigarettes a day);-He stales his alcohol consumption to be 30 units per week. On clinical
examination, significant findings include weak right foot pulses with altered sensation on
dorsum of the right toot. What is the most likely diagnosis D from the following list?

A. Autonomic neuropathy
B. Femro popliteal stenosis
C. Neuropathic joint disease
D. Polyneuropathy
E. Radiculopathy

84. A 35-year-old man presents with a three-week history of low back pain and a three day
history of pain and weakness in his left leg. Physical examination demonstrates numbness
over the posterior aspect of the left calf extending to the lateral aspect, of the foot. Left
ankle reflex is absent. A magnetic resonance scan shows compression of the left S1. nerve
root. What is the most likely structure compressing the nerve?

A. Annulus fibrosus
B Anterior longitudinal ligament
C. Nucleus pulposus
D. posterior longitudinal ligament
E Vertebral end plate

85. A 72-year-old man who is a smoker presents to his doctor with a second episode of
frank hematuria. When an ultrasound scan of his urinary tract was performed it a solid
lesion in the right kidney. What is the most likely pathology underlying this presentation?

A. Angiomyolipoma
B Renal cell carcinoma,
C Renal cyst
D Renal oncocytoma
E. Transitional cell carcinoma

86. A 20-years old man presents with abdominal pain and shock. He is found to have
hemoperotonium Due to a ruptured spleen. He denies any history of trauma what is the
most likely predisposing cause of his splenic rupture?
A. Epstein Barr virus infection
B. Human immunodeficiency virus (HIV) infection
C. Measles virus infection
D. Mumps virus infection
E. Varicella zoster virus infection

87. A 30-year-old woman presents to the outpatient' clinic witlr a three-month history of
diarrhoea, which is intermittently bloody. On rectal biopsy, the histology has shown'
a granulomatous inflammation. What is the most likely diagnosis?

A. Ameobiasis
D. Crohn's disease
C. Sarcoidosis
D. Tuberculosis
E. Ulcerative colitis

88. A 56 year-old motorcyclist presents to the Emergency Department after being


involved in a road traffic accident. He is a smoker and known to have hypertension. He is
conscious (Glasgow coma score of 1-3/15), and maintaining his own airway and breathing.
He is found to have an open right femoral fracture With normal distal pulses and-
sensation. Resuscitation is started. Urethral catheterisation drained 250 ml. immediately
but over the following 60 minutes he drained only 10 ml. Which of the following is the
most likely cause of his low urine output?

A. Blocked catheter
B. Cardiogenic shock
C. Hypovolemic shock
D. Ruptured bladder
E. Septic shock

89. A 35-year-old man is admitted with pancreatitis, which leads to admission to the
surgical high dependency unit. After 48 hours, he develops breathing difficulties with
elevated, respiratory rate and increasing hypoxia. You suspect he is developing respiratory
failure. Which of the following mechanisms is responsible for the regulation of respiration?

A Brainstem receptors producing voluntary breathing.


B CO2 concentration in medulla
C Elevated bicarbonate levels in cerebrospinal fluid
D. Hydrogen ion diffusion across blood brain barrier
E. Hypoxic stimulation' of chemoreceptors

90. A 70-year-old man undergoes transurethral resection of bladder uimonr (TURBT). The
tumour lies over the lateral side wall of the bladder just above the opening of the right
ureteric orifice. Use of the diathermy suddenly causes the patient to 'kick' because of
contraction of the hip adductors muscles. Which nerve has been stimulated?
A Femoral nerve
B Genitofemoral nerve
C Lateral femoral cutaneous nerve
D Obturator nerve
E Sciatic nerve

91. Witch of the following statements is true with relation to cardiac muscle?

A. Contraction does not involve-release of Ca 2 + by the sarcoplasmic reticulum


B. Contraction is triggered by an influx of Ca 2+ through the sacrolemma
C. Excitation is passed from one cardiac cell to another through desmoses
D. Hypertrophy is brought about by division of existing cardiac cells
E. There is capacity for regeneration

92. A 39-year-old man presents with searing left-sided loin to groin pain. Investigation.
reveals a radiodense opacity, confirmed to be in the ureter on intravenous urography.
What is the most likely composition of this man's ureteric stone?

A. Ammonium magnesium phosphate


B. Calcium oxalate
C. Cholesterol
D. Cysteine
E . Urate

93. A 25-year-oid man is playing football when he complains of posterior thigh pain whilst
sprinting with the ball. On examination he has tenderness in the lower lateral posterior
thigh. He is unable to fully extend the knee due to pain. What is the most likely injury?

A Biceps femoris tear


B. Gracilis tear
C. Gastrocnemius tear
D. Semimembranosus tear
E Semiteodinosus tear

94 . A 23 years old man is assaulted and sustains a stab injury to the left groin. The wound
is cleaned and a simple dressing applied. Six months later the patient returns-
with a pulsatile swelling in the site of the injury. The most probable lesion present is:

A. Abscess
8. False aneurysm of the femoral artery
C. lymphocele
D. mycotic aneurysm of the femoral artery
E Saphena varix
95. 12 years old girl has a diastolic murmur it is maximally audible in the 2nd left intercostal
space. Pathology of which structure is the most likely cause ?
A. Aortic valve
B. Ductus arteriosus
C Mitral valve
D. Pulmonary valve
E. Tricuspid valve

96. A 25-year-old man is admitted having been stabbed in the interior chest. On
examination he is alert and coherent. He has congested neck veins. His pulse is 140
beats/minute and his blood pressure 90/60 mmHg. He has normal breath sounds. What is
the most likely cause of his cardiac arrest 15 minutes later?

A. Cardiac tamponade
B. Congestive cardiac failure
C. Hypovolemia
D Tension pneumothorax
E Ventricular arrhythmia

97. A 20-year-old woman presents acutely with abdominal pain. Imaging reveals a right
ovarian cystic lesion which is excised. Histological examination shows a cyst lined by
keratinising stratified squamous epithelium. Areas of fat,muscle, thyroid and neural tissue
are seen in the wall. What is the appropriate pathological designation for this lesion?

A. Cystadenoma
B. Dysgerminoma.
C. Mesenchymoma,
D. Squamous cell carcinoma
E. Teratoma

98. A 21-year-old map comes into the Emergency Department with a severe headache
and neck stiffness of recent onset. You suspect acute meningitis and decide immediately
to treat with antibiotics and perform a lumbar puncture. After your third attempt to obtain
for cerebrospinal fluid (CSF) you notice that the fluid obtained is stained red. However
during the collection of CSF into three containers it is noted that the final bottle is now
cleared of any blood. Which anatomical structure is the most likely to be responsible for
this bleeding?

A.. Anterior spinal artery


B. Epidural artery
C. Subarachnoid vein
D. Veins within the erector spinae muscles
E. Vertebral venous plexus
99. A 25-year-old woman presented in the surgical clinic with a three-month history of
diarrhoea. On examination she is found to have a smooth 2 cm swelling in the front of her
neck her sister has a similar swelling which was operated on 2 years ago The swelling
moves on swallowing and the patient is clinically euthyroid Fine needle aspiration cytology
of the swelling shows amyloid stroma and serum calcitonin levels are elevated. Which of
the following is the most likely diagnosis?

A. Anaplastic thyroid carcinoma


B Follicular thyroid carcinoma
C. Medullary thyroid carcinoma
D. Metastatic thyroid carcinoma
E Papillary thyroid carcinoma.

100. A 74-year-old man with known prostatic hyperplasia presents to the Emergency
Department with acute urinary obstruction. Thirty minutes after urinary catheterization
he Collapses with a blood pressure of 74/34 mmhg a heart rate of 128 beats/minute. What
is the most likely diagnosis?

A. Cardiac failure
B. Haemorrhagic shock
C. Latex allergy
D. Septic shook
E. Vasovagal syncope

101. A 70-year-old man has a basal cell carcinoma resected from his left temple. What is
the most important prognostic indicator?

A Clinical pathological type


B. Completeness of excision
C. intensity of the lymphocytic infiltrate
D Lymphovascular space invasion
E. Mitotic index in the tumour islands

102. A 68-year-old man has undergone emergency surgery to repair a leaking aortic abdominal
aneurysm. He underwent transfusion with 6 units of packed red cells, Blood tests were performed
two hours postoperatively: haemoglobin 10.1 g/dl WBC 12.3 x 109/L , platelets 40 x109 /L, APTT
36 seconds, INR 2.2, fibrinogen 1.5 g/L, fibrin degradation products 25 mg/mi. What will these
results be consistent with?

A Disseminated intravascular coagulation


B Inactivation of Antithrombin III
C increased level of vitamin K dependent
D Plasminogen inactivation
E Protein C deficiency
103. A 46-year-old woman is seen in the pre-admission .clinic. She had a myocardial
infarction two years ago. On examination she is in atrial fibrillation and a recent. .
echocardiogram show that she had a dilated left ventricle. She is on warfarin and
her INR. (International Normalized Ratio) is 6.1 Warfarin inhibits which of the

A Antithrombin
B. Kallikrein
C. Plasminogen
D. Prothrombin
E. von Willebrand factor

104. A 29 year oId pregnant woman is having uncontrolled labor pains. She has an
epidural for pain relief. Which of the following is true?

A. An epidural puncture should only be performed below the L1 level to avoid damage to the spinal cord
B. An epidural puncture should only be performed below the L4 level to avoid the spinal cord
C.The epidural space (also known as the extradural space) contains numerous nerve roots and a venous
plexus
D. The liganientum flavum is rarely pierced during a low epidural procedure
E. The supraspinous and interspinous ligaments are rarely pierced during the procedure .

105. A 28-year-old motorcyclist presents to the Emergency Department after being


involved in a road, traffic accident. He has a closed isolated injury to his right leg.
Radiographs show a segmental fracture of the tibia and the fibula. Intramedullary nailing
of the tibia is undertaken. Six hours following the operation he complains of severe calf
pain which does not respond to analgesics. Movement of the toes severely increases the
pain. What is the most likely cause of this pain?

A. Deep vein thrombosis


B. Deep-infection
C. Embolic episode
D . Raised intra-compartmental pressure
E. Movement of the unstabilized fibula

106. A 65-year-old woman with metastatic breast cancer is admitted to hospital


confused and acutely unwell, with nausea and vomiting. Her vital signs are within
normal limits. What abnormality is most likely to contribute to her clinical
condition?
A . Hypercalcaemia
B. Hyperkalaemia
C. Hypocalcaemia
D. Hypokalemia
E Hyponatraemia
107. during recovery from varicose veins surgery 35 years old woman complains of
weakness of dorsiflexion of the ankle Physical examination reveals absent sensation over
the dorsum of the foot. Which of the following nerves is most likely
injured?
A . Common peroneal nerve
B. Deep peroneal nerve
C. Saphenous nerve
D. Superficial peroneal nerve
E. Sural nerve
108. A 78-year-old woman, with chronic obstructive pulmonary disease (COPD), becomes
confused after a right hemicolectomy. She is oh 30% oxygen and her blood gas analysis
shows:
Po2 pCO2
Finding 6.2 kPa 8.5 kPA
Normal 11.9-13.3 4.7-6.0
From what is she most likely to be suffering?

A. Severe asthma attack


B. Emphysema
C. Pulmonary embolus
D. Type 1 respiratory failure
E. Type 2 respiratory failure.

109. A 19-year-old man fell down a flight of stairs. On presentation to the Emergency
Department his Glasgow coma scale (GCS) was 14/15 with no focal neurological signs. An
hour later he vomited and his GCS deteriorated to 10. He developed a fixed and dilated left
pupil-. Which of the following accounts for the pupillary signs?

A. Compression of oculomotor nerve parasympathetic fibres


B. Compression of sympathetic fibres to the left iris
C. Compression of the afferent fibres in the left optic nerve
D. Compression of the Edinger Westphal nucleus
E. Compression of the optic chiasma

110. A 32-year-oid mountain biker presents to the outpatient clinic. He fell awkwardly,
with excessive lateral flexion of his neck to the left side. He describes some paresthesia
over his right shoulder and lateral arm. Which appears to have been improving over the
past hour. What is the most likely injury suggested by this history?

A. Axonotmesis of the C5 nerve root


B. Axonotmesis of the T1 nerve root
C. Neuropraxia of the C5 nerve root
D. Neuropraxia of the T1 nerve root
E. Neurotmesis of the C5 nerve foot
111. A 62-year-old woman presents with a firm irregular mass in the upper outer quadrant
of the right breast This appears malignant on mammography, and fine needle aspiration
cytology is reported as C5 which is the first lymph nodes to which the tumour is most likely
to metastasize?

A. Initial node.
B. Primary node -
C. Secondary node
D. Sentinel node-.
E. Virchow’s nodes
112. A 50-year-old woman presents with a history of faecal incontinence over the past, few
years. She had a prolonged and difficult first stage of labour 20 years previously. Physical
examination reveals a relatively lax anal sphincter. Which nerve is likely to have been
damaged in labour?

A. Autonomic! nerves to the rectum .


B. Genitofemoral nerve
C. Lumbosacral trunk
D. Obturator nerve
E. Pudendal nerve

113. A 26-year-old man presents to the Emergency Department with extensive bleeding
from his arm, after sustaining a glass injury.-On examination- there is n 7 cm transverse
laceration across tire anterior aspect of his elbow. On exploring the cubital fossa, you
would expect the brachial artery to be?

A. anterior to the median nerve


B. lateral to the biceps tendon.
C. lateral to the median nerve
D. medial to the median nerve
E. superficial to the bicipital aponeurosis

114. A 53-year-old man complains of 'crushing' chest pain at rest He has chronic
hypertension and is a heavy smoker. His pulse rate is 138 beats/minute and blood pressure
140/90 mmHg. What is the principal cause underlying his reduced coronary artery blood
flow?

A. Decreased cardiac muscle wall tension


B. Decreased coronary oxygen extraction
C. Decreased diastolic interval
D. Decreased systolic interval
E. Decreased vagal tone
115. A 58-year-old woman, a known patient with thoracic right-sided meningioma,
presents with features suggestive of a Brown-Sequard syndrome. The clinical,
findings will include which of the following?

A Left sided weakness, left sided proprioception and vibration loss, right sided loss of pin prick
sensation
B Right sided weakness, right sided proprioception and vibration loss, left sided loss of pin prick
sensation
C Right sided weakness, right sided proprioception and vibration loss, right sided loss of pin prick
sensation
D. Right sided weakness, left sided proprioception and vibration loss, right sided
loss of pin prick sensation
E. left sided weakness, right sided proprioception and vibration loss, left sided loss of pin prick
sensation.
116. 70-year-old woman presents with a fractured left humerus. X-rays show an osteolytic
lesion at the site of the fracture. Histological examination of a sample taken at the time of
internal fixation shows a metastatic carcinoma. Where is the most likely primary site?

A Breast
B Colon
C. Endometrium
D. Ovary
E. Stomach

117. A 74-year-old. man presents with a pulsatile swelling in his abdomen. The principal
abnormality within the wall of arterial aneurysms is loss of?

A. adventitial collagen
B. adventitial elastin
C. intimal collagen
D. intimal elastin
E medial elastin

118. A 24-year-old woman presents with a swelling in the thyroid gland. She also has
enlarged lymph nodes in the left anterior triangle of the neck. Which of the following is the
most likely diagnosis?

A. Anaplastic carcinoma
B. Follicular adenoma
C. Follicular carcinoma
D. Medullary carcinoma
E. Papillary carcinoma

119. A 20-year«okl man with a severe head injury is being ventilated using positive
pressure. which of the following is a physiological consequence of this?
A. Decreased extra fluid volume
B. Decreased cardiac preload
C. Increase in intrathoracic blood volume
D. Increased cardiac preload
E. Increased cardiac stroke volume

120. A 70-year-old man presents with a one-week history of epigastric pain and –profuse
vomiting The most likely consequence is ?

A. hypochloremic acidosis
B. hypckalaeinic alkalosis
C. raised serum chloride
D. raised urine sodium
E. reduced renin release

121. A 22-year-old men arrives in the Emergency Department with sudden breathlessness
due to a large pneumothorax. An immediate chest drain is inserted into the fifth left
intercostal space in the mid-axillary line, There is hemorrhage into the drainage bottle.
Which of the following structures is the most likely cause of this acute hemorrhage ?

A. Intercostal artery m
B. Left pericardiophrenic artery
C. Linguia of the lung
D. Right ventricle of the heart
E. Spleen

122. The right and left pulmonary arteries are derived from which of the following
embryological aortic arches?

A. Second aortic arch


B. Third.aortic arch
C. Fourth aortic arch
D. Fifth aortic arch
E. Sixth aortic arch

123. In the fetal circulation, the majority of blood passing from the inferior vena cava
into tire right atrium subsequently passes next into which; one of the following?

A. Aorta
B. Left atrium
C. left ventricle
D. pulmonary artery
E. Right ventricle
124. A 36-year-oid butcher injures his left index finger with a knife, suffering a deep
laceration on the palmar aspect of the middle phalanx. On examination he is unable to flex
DIP, which tendon is likely to have been injured?

A. flexor carpi facialis


B. flexor carpi ulnaris
C. Flexor digitorum profundus
D. Flexor digitorum superficialis
E. Flexor indicis

A 39-year-old woman with large, expanding ovarian mass that is removed surgically,
requiring a lateral pelvic wall clearance, of lymph nodes. Four days postoperatively she
complains of painful spasms in the groin area and remarks that she has an area of
numbness on the medial side of her thigh. The most likely explanation for these, problems
is that during surgery an injury was inflicted on the:

A. femoral nerve
B. genitofemoral nerve
C. lumbosacral trunk
D. L3 ventral ramus
E. Obturator nerve

126. A 70-year-old patient is found to have a sodium of 125 mmoI/L on examination


There is no signs of hypovolemia. His plasma osmolality is 280 mOsm/L and his Urine
osmolality is 1000 mOsm/L. "Which of the following is-the best explanation of these
findings?

A. Central diabetes insipidus


B. Cerebral salt wasting syndrome
C Nephrogenic diabetes insipidus .
D. Syndrome of inappropriate anti-diuretic hormone (ADK) secretion.
E. Water deprivation

127. A .surgeon is carrying out an elective splenectomy for congenital spherocytosis.


Which structure will be divided in order to mobilise the spleen from the posterior
Abdominal wall?

A. Gastrosplenic ligament
B. Lesser omentum
C. Lienoreaal ligament
D. phrenicocolic ligament
E. Short gastric vessels

128. A 45-year-old man has established post hepatic cirrhosis. At a follow-up appointment,
a palpable spleen four fingers breadths below the costal margin, is noticed. full blood
counts have shown a persistent thrombocytopenia. Bone marrow examination has shown
megakaryocytic hyperplasia. 'What is the most likely cause of the thrombocytopenia?

A. Ineffective production in bone marrow


B. Platelet destruction in the bone marrow
C Platelet destruction in the liver
D. Platelet destruction in the spleen
E. Platelet storage in the spleen

129 A 7 years old boy presents with recurrent attacks of pneumonia. Culture showed. Psudomonas
aeruginosa. His mother confirms that he had meconium ileus in the neonatal period. Which of the
following is the most likely finding in. this boy?

A. Increased sweat alanine transaminase


B Increased sweat chloride
C. increased sweat creatinine
D. Increased sweat potassium
E. Increased sweat urea

130. A 42 years old woman has a cholecystectomy and develops a self-limiting


postoperative wound infection. By what process would bacterial ingestion have
been enhanced?

A. Apoptosis
B. Autophagy
C Metaplasia
D. Opsonisation
E. Phagocytosis

131. After resection of a rectal tumour a patient experiences erectile dysfunction. Which
of the following nerves is most likely to have been damaged in surgery?

A. Genitofemoral nerve
B, Lumbosacral, plexus
C. Pelvic splanchnic nerves
D. Perinea! branch of S4
E. Pudendal nerve

132, A 15-year-old girl has been treated for tuberculosis over the previous 6 months
She presents to Her doctor with fatigue, loss of Appetite and weight loss. On examination.
her blood pressure is 90/55 mmHg and she has increased pigmentation. What is the next
most appropriate investigations

A, Aldosterone level
B Oral glucose tolerance test
C. Random Cortisol level
D ACTH stimulation test
E water deprivation test

133. A 73-year-old man with a history of irregular bowel movements presents with dysuria,
pneumaturia and an Escherichia ccli urinary tract Infection. CT scans show a mass involving
the sigmoid colon and the bladder. What is the commonest cause of this presentation?
A. Adenocarcinoma of the sigmoid colon
B Colonic diverticular disease
C Crohn's disease
D Transitional cell carcinoma of the bladder
E Ulcerative colitis

134. A 30-year-old man is assaulted with a hammer and sustains a depressed


fracture at the Vertex, of the skull Profuse venous bleeding is noted at the fracture site. Which
vascular structure is most likely to have been perforated?

A. Cavernous sinus
B. Inferior sagittal sinus
C. Sigmoid sinus
D. Superior petrosal sinus
E. Superior sagittal sinus

135. A 57-year-old heart transplant recipient is keen-to join the cardiac, rehabilitation
programme Which of the following factors is most likely to increase cardiac output in this
patient during moderate exercise?

A. Decreased negative intrathoracic pressure


B. Decreased venous tone
C Decreased ventricular compliance
D Increased atrial filling
E. Increased intrapericardial Pressure

136. An 18-year-old woman fell 9 meters while rock climbing. She had a renal transplant
two years ago. She is brought to the Emergency Department for resuscitation and
appropriate analgesia given. When correctly prescribed, witch of following analgesic drugs
would be most .likely to adversely affect her renal function in this situation?

A. Diclofenac
B. Dihyclrocodeine
C. Morphine
D. paractamol
E. Tramadol

137. A 77-year-old man is admitted with acute urinary retention secondary to benign
prostatic hyperplasia. A urinary catheter is inserted and ot has free flow of
concentrated urine. Which of the following best accounts for the action of the
counter current multiplier mechanism that leads to the concentration of urine in the
Loop of Henle?
A. Active transport of solute out of thin section ascending limb
B. Impermeability to water in thick ascending limb
C. Permeability to solute in descending limb
D. Permeability to solute in thick ascending limb
E. Permeability to water of thin section ascending limb;

138- A 52-year-old man is found to have multiple myeloma, What skull X-ray appearances
would be characteristic?

A. Diffuse thickening of the calvarium


B. Hair on end* appearance
C. Multiple fractures
D. Multiple osteolytic lesions
E. Multiple osteosclerotic lesions

139. A 56-year-old man undergoes resection of his ileum secondary to ischemic bowel
Which of the following effects is most likely to be seen in this patient postoperatively?

A. Decreased bile salt synthesis by the liver


B. Decreased concentration of bile salts in colonic content?
C. Decreased mean corpuscular volume
D. Decreased reabsorption of vitamin B12
E. Decreased stool frequency

140. A previously healthy 45 years old woman undergoes emergency splenectomy


following traumatic rupture of the spleen. A left subclavian central venous pressure (CVP)
line to guide fluid therapy is inserted. At this time she becomes distressed and acutely
dyspneic, with a blood pressure of 75/40 mmHg. There are decreased breath sounds on the
left. Her temperature is 35.5°C. What is the most likely cause?

A. Acute blood Loss


B. Anaphylaxis
C. Myocardial infarction
D. Sputum retention
E. Tension pneumothorax

141. During the second (proliferative) phase of wound healing the predominant cells in
the wound site are fibroblasts. This cell is of mesenchymal origin -and produces the
matrix and collagen needed; to strengthen the scar. Cross linkage of collagen requires
hydroxy-proline and hydroxy-lysine residues, which requires a specific vitamin to be
available in sufficient quantities. Deficiency of which vitamin results in collagen that is
unstable?
A. Vitamin B2 (riboflaVine)
B. Vitamin B6 (pyridoxine)
C. Vitamin C (ascorbic acid)
D. Vitamin D (cholecalciferol)
E. Vitamin E (tocopherol)

142. A 19- year-old man fell down a flight of stairs. On presentation to the Emergency
Department his Glasgow Coma Score (GCS) was 14/15 with no local neurological signs. An
hour later he Vomited and his GCS deteriorated to 10. He developed a fixed and dilated
left pupil. Which of the following is likely to seen on the scan?

A. Left-sided acute subdural haematoma


B. Left-sided extradural haematoma
C. Left-sided midbrain haematoma
D. Right-sided acute subdural haematoma
E. Right-sided extradural haematoma

143. A 70-year-old woman weighing 5Q kg undergoes open cholecystectomy.


Postoperative pain relief is administered by intravenous morphine infusion (morphine 1
mg; 5 minute lock out).-Twenty-four hours postoperatively she is in an agitated, confused
state. Her temperature is 37.1°C-.pulse 90 beats/minute 110/80 mmHg. Her urine output is
35 ml/hour. Arterial blood gases are
PH pO2 pCo2 Bicarbonate Base Excess
Finding 7.27 9.6 kPa 10.0 kPA 29 mmol/L -6
Normal 7.35-7.45 11.9-13.3 4.7-6.0 22-26 -2 to +2
Which of the following is the most likely explanation to account for false findings?

A. Hypovolaemia
B. Opiate induced hypoventilation
C. Pain induced hyperventilation
D. Pulmonary embolus
E. Silent myocardial infarction

144. A 70-year-old woman from a nursing home is brought to' the Emergency Department
with abdominal pain and vomiting. On examination, she is dehydrated and her abdomen is
distended. There is a 3 cm x 4 cm swelling in the right groin which is nontender, and there
is no cough impulse. At operation, a femoral hernia is found. Which one of the following
lies immediately lateral to the hernial neck?
A. Femoral artery
B. Femoral nerve
C. Femoral vein
D. Inguinal ligament
E. Pectineal ligament
145. A 64-year-old woman is-reviewed in the Emergency Department with an acute
right groin swelling, below and lateral to the pubic tubercle. Which of the following forms
the medial boundary of the ring through which this hernia is protruding?

A. Conjoint tendon
B. Inferior epigastric artery
C. Inguinal ligament
D. Lacunar ligament
E. Pectineal ligament

146, A. 27-year-old has had pancolitis for the last five years. Which one of these
complications is he least likely to develop?

A. Arthritis
B. Cholangitis
C. iritis
D. Polyarteritis
E. Toxic megacolon

147. An 82-year-oid woman had a hemiarthroplasty of the hip six hours ago! She is an
insulin-dependent diabetic and has been on long-term non-steroidal analgesics ,In the last
three hours she passed,, via her urinary catheter, a total of 20 ml of urine. Her pulse rate is
120 beats/minute and regular, her blood, pressure 100/60 mmHg and capillary refill is 3
seconds. Blood urea and electrolyte levels are:

Sodium Potassium Creatinine Urea


Finding 135 mmol/L 4.0 mmol/L 140 umol/L 12.0 mmol/L
Normal 137-144 3.5-4.9 35-110 3.2-7.5
The most likely cause of this is:
A. Acute Tubular necrosis
B. blocked urinary catheter
C. diabetic nephropathy
D. non-steroidal: anti-inflammatory drugs
E. Reduced renal perfusion

148. Witch single respiratory test parameter (FEVJ Forced expiratory volume in one
second; FVC - Forced vital capacity) is the best prediction for morbidity and mortality after
surgery?

A. Confronted flow volume loop


B. FEV1 < 1L
C. FEV1/FVC <70%. .
D. Maximum inspiratory Volume < 2 L
E. Reduced residual volume
149. A 58-year-old man, admitted for inguinal hernia surgery, is found to have an enlarged
heart shadow on his chest radiograph. Which one of the/following statements is true of the
right ventricle?

A. Has a muscular wall that is normally thicker than that of the Left ventricle
B. Forms the apex of the heart
C. Forms the right border of the heart
D. Is directly related to the diaphragm
E. Gives rise to the aorta

150. A 67-year-oid man is admitted to hospital for investigation of at irregular heart beat.
The 'conducting system' of the heart includes which one of the following anatomical
structures?

A. Atrioventricular node
B. Chordae tendineae
C. Fibrous skeleton of the heart
D. Interatrial septum
E. Phrenic nerve

151 A 28-year-old man is found in the sea by the emergency services. he is alert but
confused. His core temperature is below 33°C. ECG changes that may occur include which
one of the following?

A. Atrial flutter
B. inverted T wave at the junction of the QRS & T wave
C. Narrowed QRS complex
D. Prolonged PR interval
E. Shortened QT interval

152. A 75-year-old woman presents to the outpatients clinic with a three-months history
of painless ulcer on the sole of her left foot. On examination she is obese" with a 1.5 cm
deep ulcer over the first metatarsal head. Her foot is warm and pedal pulsesare palpable.
She has a hemoglobin of 13 g/dL, normal urea and electrolyte level. What is the most likely
cause of her ulcer?
A. Atherosclerosis
B. Diabetic neuropathy
C. Embolism
D. venous hypertension
E. Vasculitis
153. A 24-year-old man is assaulted and sustains a deep 2 cm long laceration to the lateral
aspect of his lower leg. Physical examination show's that the laceration is anterior to the
lateral malleolus. Which of the following structures is most at risk of
being divided?

A. Peroneus brevis
B. Peroneus longus
C. Saphenous nerve
D. Superficial peroneal nerve
E. Sural nerve

154. A 48-year-old woman presented to the Emergency Department with persistent


diarrhoea. On examination, she was apyrexial and had an irregularly irregular pulse:
of 140 beats/minute. The most likely cause of the abnormal cardiac rhythm is:

A. hypernatraemia
B. hypokalaemia
C. hypovolemia
D. myocardial ischemia
E. sepsis

155. A 45 years old woman is investigated after two episodes of renal colic. A plain
abdominal X-ray reveals small opacities in both kidneys. No abnormalities are found on
physical examination. Biochemical investigations show:
Findings Normal
Adjusted ( corrected) Calcium 2.85 mmol/L 2.15-2.55 mmol/L
Phosphate 0.70 mmol/L 0.6-1.25
Alkaline phosphatase 195 IU/L 20-120
Creatinine 72 umol/L 35-110
PTH 12.7 pmol/L 0.95-5.7
Which, of the following is the most likely cause?
A. Primary hyperparathyroidism
B. Primary hypoparathyroidism
C. Pseudohyperparathyroidism
D. Secondary hyperparathyroidism
E. Tertiary hyperparathyroidism

156. A 35-year-cld man develops septicaemia following surgery for perforated acute
appendicitis. He is hypotensive. Arterial blood-gas analysis reveals
PH pO2 pCo2 Bicarbonate
Finding 7.25 7.2 kPa 7.5 kPA 17 mmol/L
Normal 7.35-7.45 11.9-13.3 4.7-6.0 22-26
Which of the following is the most likely explanation to these findings
A. Compensated metabolic acidosis
B. Compensated respiratory acidosis
C. Mixed metabolic and respiratory acidosis
D. Uncompensated metabolic acidosis
E. Uncompensated respiratory acidosis

157. A 75-year-old man is assessed for surgery. he has smoked 25 cigarettes a day for -
the past 45 years. He is short of breath on minimal exertion. Chest: auscultations reveals a
widespread wheeze. A chest X-ray shows hyperinfilated lungs. An arterial blood gas
sample is likely to reveal which of the following changes?

pCO2 Bicarbonate
A High High
B High Low
C Low High
D Low Low
E Normal Normal

158. An 18-year-old woman present with recurrent tonsillitis and spontaneous skin
bruising. On examination there is no lymphadenopathy or splenomegaly. What is the most
likely diagnosis?
A. Acute myelocytic leukemia
B. Agranulocytosis
C. Aplastic anemia
D. Hodgkin's disease
E. Human immune deficiency virus infection

159. A 62-year old woman presents with a firm "irregular mass in-the upper outer quadrant
of the right breast. This is shown to be malignant on mammography and fine needle
aspiration-cytology. She is treated with wide local excision and axillary clearance. Which
of the following histopathological findings would imply a better prognosis?
A. Absence of HER2 amplification
B. Bloom and Richardson Grade III
C. More than three positive axillary nodes
D. Presence of lymphovascular invasion
E. Presence of estrogen receptors

160. A.58 year-old man is due to have surgery for Dupuytrens contracture to release
the ring finger of the light hand. 'Which one of the following regarding the anatomy
of the ring finger is correct?
A. Has a cutaneous innervation from the radial nerve
B. Has no cutaneous innervation from the median nerve
C. Has no cutaneous innervation from the ulnar nerve
D. Has the flexor digitorum profundus inserting into the middle phalanx
E. Is abducted by a palmar interosseous muscle
161. A 75-year-old man presents with deep vein thrombosis. Computerized Tomography
confirms the presence of a thrombus' in His common iliac vein extending into the
inferior vena cava The common iliac veins join to form the inferior vena cava at
which vertebral level?

A. L3
B. L4
C. L5
D. S1
E. S2

162. A 35-year-old motorcyclist is involved in a road traffic accident and fractures his
temporal bone, completely transecting the facial nerve just distal to the geniculate
ganglion. Which function of this nerve remains?

A. Lacrimation
B. Salivation from the submandibular gland
C. Sensation to the anterior tongue
D. Smile
E. Taste

163. A 30-year-okl motorcyclist is brought to the Emergency Department after a road traffic
accident. He has an open fracture of his right femur, On arrival he-is tachypnoeic and confused,
with cold and clammy skin. Which of the following physiological changes is most likely to be seen?

A. Decreased reabsorption of water from, the collecting tubules


B. Decreased serum bicarbonate level
C. Increased pH of blood
D. increased secretion of sodium in the urine
E. Increased synthesis of glycogen in the liver

164. A 50-year-old man on the waiting list for a hip replacement attends the preoperative
clinic. He has a body mass index (SMI) of 41 and a risk of perioperative hypoxia. Reduction
of which of the following volumes or capacities is this risk most likely to be due to?

A. Expiratory reserve, volume


B. Functional residual capacity
C. Inspiratory reserve volume
D. Total lung volume
E. Vital capacity

165. A 4-year-old boy presents to the Emergency Department with a two-day history of
Headache, vomiting and drowsiness. A. CT scan reveals dilatation of both his lateral
ventricles and his third cerebral ventricle: His fourth ventricle was of normal size. It is
suspected that he has an obstruction to his cerebrospinal fluid flow At which of the
following sites is the obstruction most likely to be?

A. Cerebral aqueduct of Sylvius


B. Infundibular recess
C. Interventricular foramen (of Monro)
D. Lateral foramen of fourth ventricle (foramen of Luschka)
E. Median foramen of fourth ventricle (foramen of Magedie)

166. A 74-ysar-old man, who has undergone emergency major abdominal surgery two
days previously, is noted to be confused. He has been on furosemide for mild heart failure;. The
plasma sodium is 122 mmol/L. Inspection of the fluid chart shows that he has been written up for
four-hourly intravenous 5% glucose infusions. What is the most likely cause for the hyponatremia?

A. An ACTH (Adrenocorticotrophic hormone) response to surgery


B. Excessive intravenous fluid administration
C. Osmotic effect of hyperglycemia induced by glucose infusions
D. Syndrome of inappropriate antidiuretic hormone
E. Use of loop diuretic in the long term

167. During arch aortography, the vertebral artery would be seen to; a rise directly from
which of the following?

A. Arch of the aorta


B. Brachiocephalic artery
C. Common carotid artery
D. Internal carotid-artery
E. Subclavian artery

168. A 60-year-old non-smoker presents with a three-month history: of loss of weight,


malaise and breathlessness. On examination, he has left supraclavicular lymph node
enlargement! Chest radiograph reveals multiple bilateral opacities. 'What will biopsy of left
supraclavicular lymph nodes most probably show?

A. Chronic inflammatory cells


B Granuloma
C. Langerhan's type giant cells
D. Multiple abnormal mitotic figures in cells
E. Reed Stenberg cells
169. A 12-year-old child was admitted to the Emergency Department having inhaled a
peanut. Where will the peanut most likely to have lodged?

A. Left lower lobe bronchus


B. left upper lobe bronchus
C. Right lower lobe bronchus
D. Right middle lobe bronchus
E. Right upper lobe bronchus
171. A 39 years old multiparous woman who smokes 10 cigarettes per day presents with
2cm x 2 cm tender lump in her right breast. There is a brown discharge from the nipple.
Mammography is unhelpful. And ultrasound shows some dilated lactation ducts What is
the most likely pathological diagnosis?

A. Breast abscess
B. Fat. Necrosis
C. Fibrocystic disease
D. Phylloides tumor
E. Plasma ceil mastitis

172. A-56 year oId man presents with headache and vomiting with deterioration over the
last two weeks. On examination he has papilledema. A CT scan shows a space occupying
lesion in the right parietal lobe. He develops a sudden homonymous hemianopia. Which
past of the visual system to be affected?

A. Lens
B. Optic Chiasm
C. Optic radiation
D. Retina
E. Second cranial nerve

173. Atrial depolarization is shown by which ECG change?

A. P
B. Q
C. R
D. S
E. T
174. A 65 years old man with known ischemic heart disease and atrial fibrillation presents
with a paralysis of his right lower limb. Which arterial territory is most likely to be
involved?

A. Left anterior cerebral


B. Left middle cerebral
C. right anterior cerebral
D. Right middle cerebral
E. Right posterior cerebral
175. A 91 year old man becomes hypotensive two days after THR. He is found to be in atrial
fibrillation with a pulse of 140 beats/minute. After treatment with digoxin his AF slows and
he has a pulse rate of 90 BPM. Which form of the venous wave form is absent?

A. A wave
B. C wave
C. V wave
D. A decent
E. Y decent

176. A 56-year-old man with chronic emphysema is on the high dependency unit (HDU),
ten days after anterior resection. He has developed acute shortness of breath' and
hypotension): Which of the following landmarks would be the most appropriate to use to
obtain a femoral arterial blood gas sample?

A. A point midway between the anterior superior iliac spine and the pubic tubercle
B. A point midway between the anterior superior iliac-spine and the' pubic symphysis
C. A point midway between the greater trochanter and the pubic symphysis
D. a point 1 cm inferior and 4 cm lateral to the pubic tubercle
E. A point 2 cm lateral to the midway point between the anterior superior iliac spine and
pubic symphysis

177 . A 4S-year-old woman with insulin-dependent diabetes attends to the ER with abdominal pain
and confusion. She has been treated for three days with antibiotics for chest infection. The
creatinine is 400umol/L and her blood sugar 42 mmol/L. The most likely severe electrolyte
abnormality is:
A. Hypercalcemia
B. Hyperkalemia
C. Hypermagnesemia
D. Hypocalcemia
E. Hypomagnemsia

End of paper 2

Vous aimerez peut-être aussi